Почему детонирует двигатель после выключения зажигания: Что делать, если двигатель детонирует после выключения зажигания?

Содержание

причины, как исправить, чем грозит

Детонация в двигателе после выключения мотора — крайне неприятная проблема, с которой может столкнуться любой водитель. Причиной детонации является самовозгорание топлива после завершения работы мотора, а последствия от такой проблемы могут быть самые разные, вплоть до повреждения ключевых элементов двигателя и необходимости проведения капитального ремонта. В рамках данной статьи рассмотрим, что делать, если после выключения зажигания происходит детонация.


Оглавление: 
1. Чем грозит самопроизвольная детонация
2. Как диагностировать самопроизвольную детонацию
3. Почему возникает самопроизвольная детонация после выключения двигателя
4. Что делать при самопроизвольной детонации

Чем грозит самопроизвольная детонация

Перед тем как разбираться с причинами проблемы, важно более подробно ознакомиться с ситуацией. Самопроизвольная детонация — это крайне нежелательный процесс для любого двигателя.

При возгорании и взрыве топлива вне времени работы мотора происходит мощное физическое воздействие на всю цилиндро-поршневую группу. Соответственно, в результате такой детонации наносится вред цилиндрам, поршням, шатунам, коленвалу и другим деталям.

Важно: Нагрузки в момент самопроизвольной детонации после выключения мотора достаточно высокие, и со временем они навредят работе мотора в любом случае, поэтому важно выявить и устранить проблему на ранних этапах.

Как диагностировать самопроизвольную детонацию

Определить, что после выключения зажигания в двигателе происходит детонация, очень просто. На это указывает соответствующий звук.

Обратите внимание: Самопроизвольная детонация после выключения двигателя — это не моментальная реакция. Процесс длится на протяжении 20-30 секунд, в течение которых слышны хлопки со стороны двигателя.

Многие неопытные водители считают, что подобная ситуация с самопроизвольной детонацией является нормальной.

На самом деле это не так. После выключения зажигания никаких посторонних двигателей от мотора не должно исходить.

Важно: Если вы диагностировали, что у вас после выключения двигателя автомобиля происходит в нем самопроизвольная детонация, не следует затягивать с решением проблемы. Обязательно самостоятельно установите причину такой неисправности устраните ее, либо обратитесь в сервисный центр. Если вовремя не предпринять шаги для устранения самопроизвольной детонации после выключения двигателя, это может привести к серьезным проблемам в работе мотора.

Почему возникает самопроизвольная детонация после выключения двигателя

Можно выделить две основные причины возникновения самодетонации после выключения зажигания:

  • Использование топлива, которое не предназначено для данного автомобиля. Речь идет, например, о бензине с более низким октановым числом, чем рекомендует производитель. Бензин с низким октановым числом предполагается использовать на моторах, степень сжатия в которых невысока.
    Если, например, для двигателя рекомендуется бензин АИ-95, а в него налить топливо АИ-80, то оно будет самопроизвольно детонировать;
  • Неправильно выставленное зажигание. Чтобы топливо детонировало, зажигание выставляется ранним. Это приводит к тому, что возгорание бензина происходит в момент движения поршня на сжатие. Соответственно, возникает перегрев рабочего пространства двигателя. Из-за повышенной температуры детонации происходят непроизвольно и после выключения зажигания.

Это наиболее частые причины, почему возникает самопроизвольная детонация топлива. Но также нельзя исключать вариант, что проблема кроется в свечах. Если используются свечи, которые не рекомендуют производителем, может возникать такая проблема.

Что делать при самопроизвольной детонации

Если вы замечаете, что после выключения двигателя у вас в моторе происходит самопроизвольная детонация, рекомендуется:

  1. Убедиться по технической литературе к автомобилю, что используется рекомендованное производителем топливо. Возможно, потребуется сменить заправку на более вызывающую доверие, способную предоставить топливо лучшего качества;
  2. Далее проверьте, как выставлено зажигание. Если установлено раннее зажигание, настройте его ближе к средним значениям;
  3. Проверьте свечи зажигания, обратите внимание на нагар на них. Используйте только свечи, которые рекомендует производитель.

Самопроизвольная детонация — крайне неприятное явление, которое следует устранить сразу после обнаружения первых симптомов.

Загрузка…

4 причины и 5 последствий детонации — журнал За рулем

Пришла весна — самое время прохватить на хорошей скорости. И услышать звонкие постукивания в бензиновом двигателе, поднакопившем за зиму нагара… Так вот ты какая, детонация!

КАК ЗВУЧИТ ДЕТОНАЦИЯ?

Звук детонации напоминает частые звонкие удары по блоку цилиндров, примерно как если бы по нему стучали гаечным ключом среднего размера. Частота пропорциональна оборотам коленвала.

Чаще всего детонация происходит в одном, самом нагретом цилиндре. На шоферском жаргоне прошлых лет детонацию называли звоном или стуком пальцев — но никакого отношения к поршневым пальцам природа возникновения звука не имеет.

Материалы по теме

Чем опасна?

Двигатель, работающий с сильной детонацией и большой нагрузкой, выходит из строя за считаные минуты. Повреждение вызывают как механические напряжения, так и сильный перегрев деталей.

Материалы по теме

  • Чаще всего страдает поршень — деталь, не имеющая непосредственного теплоотвода и изготовленная из сплава со сравнительно низкой температурой плавления.
  • Разрушаются перегородки между поршневыми кольцами.
  • Возможно подгорание и растрескивание тарелок клапанов, иногда наблюдается прогорание прокладки головки блока цилиндров.
  • Порой страдают свечи зажигания.
  • Детонация вызывает вибрацию двигателя, что ухудшает смазку трущихся поверхностей и даже может приводить к разрушению поршневых пальцев и шатунных вкладышей.

Как должно быть?

Рабочая смесь воспламеняется от свечи зажигания, после чего фронт пламени распространяется в камере сгорания со средней скоростью 20–30 м/с. Это сопоставимо со средней скоростью поршня на номинальных оборотах, составляющей обычно около 15 м/с. ­Поэтому горение распространяется от свечи не в виде идеальной полусферы. Большое влияние оказывают завихрения топливовоздушной смеси в цилиндре, которые при конструировании стараются сделать максимально мощными.

А как бывает?

Иногда спокойное, относительно медленное горение смеси превращается в быстрое и взрывообразное — детонацию. Резко увеличивается давление и растет плотность смеси — так возникает ударная волна. Отсюда и самое короткое определение детонации: это процесс сгорания, идущий во фронте ударной волны.

Материалы по теме

Толщина фронта соответствует всего нескольким длинам свободного пробега молекул. Резкое выделение энергии приводит к возбуждению рядом расположенных молекул, а потому распространение процесса идет очень быстро — со скоростью более 2000 м/с.

Мгновенное повышение температуры газа в ударной волне вызывает взрывную реакцию, энергия которой поддерживает распространение волны. Когда эта волна — или волны, если мест самовоспламенения несколько — достигает поверхностей камеры сгорания, появляется характерный металлический стук.

При нормальной работе мотора фронт сгорания повышает давление в цилиндре — собственно, он на это и рассчитан. Он сжимает оставшуюся смесь до 50–60 бар, температура при этом составляет примерно 300˚ С. Если эти параметры превышены, то может возникнуть очаг детонации. Однако эти же параметры должны быть возможно бóльшими для повышения эффективности работы двигателя. Поэтому оптимально настроенным двигателем считается такой, в котором сгорание завершается на грани детонации.

Основные причины детонации

Материалы по теме

  • Применение топлива, октановое число которого ниже рекомендованного производителем автомобиля.
    Тут возможны два варианта: либо владелец от жадности заливает, например, АИ‑92 вместо АИ‑95, либо его обжулили на АЗС.
  • Мотор неверно отрегулирован. Чаще такое встречалось на карбюраторных машинах, в которых легко было сбить угол опережения зажигания, разрегулировать состав топливной смеси и т. п. Наиболее склонна к детонации обедненная топливная смесь (при коэффициенте избытка ­воздуха α = 1,1 вместо единицы).
  • Степень сжатия повышена вследствие неумелого ремонта — фрезерования блока цилиндров или головки, установки тонкой прокладки.
  • Изношенность двигателя. Детонацию может спровоцировать моторное масло, попавшее в камеру сгорания, или нагар, накопившийся после зимы.

Детонационные разрушения поршня.

Детонационные разрушения поршня.

Когда бывает детонация

  • На очень малых оборотах — например, при парковке в жару хорошо прогретого автомобиля с ручной коробкой.
  • Когда мотору очень жарко: вы долго протолкались в пробке, после чего наконец-то дали интенсивный разгон.
  • При большой нагрузке на двигатель, например, при подъеме в гору на высокой передаче.

Заметьте, что любая автоматическая коробка передач облегчает жизнь мотора, не допуская его работы на низких оборотах, когда в процессе горения смеси хватает времени, чтобы образовался очаг самовоспламенения.

Что делать?

Сгладить остроту проблемы позволило повсеместное применение датчиков детонации. Они реагируют на высокочастотные колебания блока цилиндров, возникающие при детонационном сгорании. Пьезокерамический чувствительный элемент создает сигнал переменного напряжения. Когда его амплитуда и частота показывают, что пошла вибрация стенки блока цилиндров, блок управления корректирует угол опережения зажигания в сторону более позднего, а также параметры подачи топлива. Обычно датчик детонации устанавливают на наружной стенке блока цилиндров в середине, а если двигатель V‑образный, то на каждом ряду цилиндров.

Калильное зажигание и дизелинг

Материалы по теме

Иногда за детонацию ошибочно принимают другие явления.

При «калильном зажигании» воспламенение происходит не от искры свечи зажигания, а от перегретой зоны в камере сгорания. Виноватыми могут быть неверно подобранные свечи или частицы нагара. Недаром же главной характеристикой свечи является калильное число, то есть способность отводить тепло от электродов и изолятора.

Другое явление — «дизелинг», то есть работа мотора после выключения зажигания, происходит от сжатия рабочей смеси в сильно разогретом моторе. Калильное зажигание носит устойчивый характер, «дизелинг» — кратковременный. Бороться со вторым намного проще: достаточно «отрубить» подачу топлива после выключения зажигания, как и сделано на всех современных моторах.

ДЕТОНАЦИЯ И… МУЗЫКА!

В магнитофонную эпоху все любители музыки знали — нет дефекта противнее детонации! Так называли искажение звука в результате модуляции посторонним сигналом в диапазоне частот от 0,2 до 200 Гц. Вследствие неоднородного движения магнитной ленты звук как бы плавал — в литературе термину детонация эквивалентен составной термин wow and flutter (где wow — «медленная» детонация, или «плавание» звука, а flutter — «быстрая»).

А еще детонацией называли фальшивое пение (от фр. detonner — «петь фальшиво»), при котором звук то и дело отклонялся от нужной высоты.

Как избежать детонации?

Материалы по теме

Главное правило — никогда не заправляться бензином с пониженным октановым числом. Инженеры проектируют двигатели с определенным запасом, учитывая то, что реальное октановое число может оказаться чуть ниже заявленного. Поэтому кратковременная езда на 92‑м вместо 95‑го, как правило, вреда не приносит. Но если заливать 92‑й постоянно, то вместо него однажды можно нарваться на условный «89‑й», и это уже будет смертельно.

Ну а если двигатель детонирует даже на заведомо нормальном бензине, не откладывайте визит на сервис.

  • На каких современных авто можно проехать 500 000+ км? Все семь моделей — тут.
  • Некачественный бензин, бесконечные путешествия по пробкам, постоянные перегревы мотора приводят к быстрому износу свечей зажигания. Проверяйте их чаще и меняйте по мере необходимости.
  • Всегда в продаже специальная и техническая литература, выпущенная издательством «За рулем».

Детонация двигателя после выключения зажигания

Детонация двигателя может произойти в каждом автомобиле. Многие водители сталкивались с этой проблемой и не знали как действовать в такой ситуации. Ниже в статье будет рассмотрена проблема детонации двигателя после выключения зажигания, а также ее причины возникновения и варианты ремонта.

Содержание статьи

Что такое процесс детонации и чем он может быть опасен

Детонация двигателя представляет собой явление, которое называется процессом возгорания смеси в цилиндрах двигателя. Выглядит так: когда в момент нормальной работы скорость распространения пламени равняется 30 м/с, то в момент детонации скорость составляет до 2000 м/с.

Узнать, что в моторе начался этот процесс можно по звуку (металлические стуки), который исходит из-под капота.

В момент детонации происходит нарушение масляного слоя, что в свою очередь приводит к тому, что головка блока цилиндров деформируется, а это может спровоцировать поломку ДВС.

Помимо этого из-за критически повышенной температуры есть возможность перегорания прокладки ГДР и появление ржавчины на дне поршня. Худшее к чему может привести детонация – это полная замена цилиндров, коленвала, головки блока и поршневой группы.

Причины детонации

Причины, которые являются провокаторами появления этой поломки можно образно разделить на такие группы:

  1. Количество октана в топливе;
  2. Конструкционные факторы ДВС;
  3. Эксплуатация машины.

Бензиновые моторы больше подвержены детонации, нежели дизельные. Главным показателем бензина является число октана в нем, и чем оно ниже, тем выше шансы детонации. Если авто заправить не привычным для него 95, а 92 бензином, то есть большой шанс что при высоких нагрузках топливо может детонировать. Но также проблема может скрываться и в некачественном топливе. Для того чтобы повысить октановое число, недобросовестные люди добавляют в низкокачественный бензин газ (бутан или пропан).

Этот газ испаряется и остается плохой бензин, на котором работает и от которого «умирает» двигатель. Детонация после выключения зажигания происходит именно по той причине, что в камере сгорания, на ее стенках, образуется нагар от низкооктанового топлива. В результате, когда произошло выключение зажигания, двигатель продолжает работать по той причине, что смесь воспламеняется от раскаленных деталей на свечах или от нагара.

К причинам конструктивным, провоцирующих появление детонации, можно отнести форму камеры сгорания, наличие наддува, месторасположение свечей зажигания, степень сжатия и днище поршня.

Если высокая степень сжатия, то ДВС имеет склонность в детонации. Если присутствует система наддува, то для таких машин необходим бензин с высоким октановым числом.

Эксплуатация авто не меньшее влияет на появление этой проблемы. Если машина работает на повышенной передаче с маленькой скоростью или на гору подниматься на пятой со скоростью 20 км/ч то это идеальные условия для появления детонации.

Как бороться

Каждый водитель должен понимать, что основной причиной детонации является топливо, а точнее – некачественное топливо. Поэтому при появлении этой проблемы необходимо первым делом проверить бензин. При необходимости заправляйте авто у проверенных поставщиков. Знайте, чем дешевле топливо, тем велик шанс, что оно абсолютно не качественное. Иногда лучше переплатить символическую сумму, чтобы завтра не столкнуться с проблемой.

Также нужно отрегулировать зажигание, если это необходимо. Если при раннем зажигании ничего не меняется, попробуйте еще раз его отрегулировать, поставьте в нейтральное положение. Возникновение также возможно и при небольших нагрузках, но при длительной эксплуатации. В этом случае на стенках цилиндра образуется слой нагара, из-за которого происходит увеличение сжатия и снижение отвода тепла. Чтобы такого не происходило, желательно проводить диагностику: хотя бы один раз в месяц давать мотору нагрузку (разогнать машину до высокой скорости на несколько минут). Желательно это делать за пределами города.

Необходимо также проверить свечи зажигания. Но тут нужно помнить, что свечи для вашего автомобиля были рекомендованы производителем машины, поэтому тут нужно прислушиваться к их мнению. В противном случае, вам вскоре придется опять возвращаться к ремонту автомобиля.

Иногда детонация может сопровождаться зеленым или черным потоком дыма из выхлопной трубы. Это свидетельствует о том, что поршни разрушаются или уже разрушились и происходит выброс частей алюминия черед эту трубу. В данном случае, регулировка не поможет. Тут нужно заняться заменой всей поршневой группы.

Итак, вы проверили каждую возможность, и все равно не смогли выяснить, откуда растут ноги? Иногда случается так, что причиной может быть перегрев двигателя. Вам нужно проверить жидкость для охлаждения и выяснить состояние термостата. В том случае, когда и тут все в порядке, лучшим советом будет отправиться в ближайшую станцию технического обслуживания.

Хочется думать, что в этой статье каждый узнал что-то новое и полезное для себя. Чтобы никогда не сталкиваться с подобной проблемой берегите своего коня, не заправляйте его на непроверенных АЗС и иногда делайте диагностику всех его систем и узлов. Удачи!

Видео “Причины и последствия детонации мотора”

На записи эксперт рассказывает о причинах и последствиях детонации мотора автомобиля.

Двигатель после выключения зажигания некоторое время продолжает кратковременно работать («дергается»)[3]

Читайте также

Двигатель не запускается в холодное время года

Двигатель не запускается в холодное время года Неисправности системы пуска Якорь стартера не вращается при включении замка зажигания (см. пояснения на стр. 4). При очень низких температурах (-30…-35 °C) двигатель не запускается. Пуск двигателя можно осуществить путем

После сбрасывания газа двигатель глохнет

После сбрасывания газа двигатель глохнет Неисправности карбюратора Не отрегулирован холостой ход. На исправном и проверенном по основным регулировочным параметрам прогретом двигателе винтами качества и количества смеси добиться устойчивой работы двигателя с

Свечи зажигания

Свечи зажигания Сколько служит свеча, каков ее ресурс, то есть период 100-процентной надежности?Надежная свеча, от которой зависит эффективность поджигания рабочей смеси в цилиндрах двигателя, может эксплуатироваться долго, если периодически очищать ее электроды от

12.

3. Работать до восьмидесяти лет?

12.3. Работать до восьмидесяти лет? Женевьев Ридей-Мальви, работающая с Джиарини и Штаэлем, заметила важное обстоятельство, которое привело ее к столь же важному предложению. Она говорит, что экономика услуг предоставляет больше возможностей для работы людям с богатым

6.6.7. ПОЛУПРОВОДНИКОВЫЕ ПРИБОРЫ В ЭЛЕКТРОПРИВОДЕ. СИСТЕМЫ ТИРИСТОРНЫЙ ПРЕОБРАЗОВАТЕЛЬ — ДВИГАТЕЛЬ (ТП — Д) И ИСТОЧНИК ТОКА — ДВИГАТЕЛЬ (ИТ — Д)

6.6.7. ПОЛУПРОВОДНИКОВЫЕ ПРИБОРЫ В ЭЛЕКТРОПРИВОДЕ. СИСТЕМЫ ТИРИСТОРНЫЙ ПРЕОБРАЗОВАТЕЛЬ — ДВИГАТЕЛЬ (ТП — Д) И ИСТОЧНИК ТОКА — ДВИГАТЕЛЬ (ИТ — Д) В послевоенные годы в ведущих лабораториях мира произошел прорыв в области силовой электроники, кардинально изменивший многие

8.3.1. СИСТЕМЫ ЗАЖИГАНИЯ

8.3.1. СИСТЕМЫ ЗАЖИГАНИЯ Низковольтная магнитоэлектрическая машина, названная впоследствии «магнето низкого напряжения», была впервые применена для зажигания двигателей внутреннего сгорания (ДВС) в 1875 г. От магнето осуществлялось зажигание на отрыв — внутри цилиндра ДВС

Грузовые автомобили. Система зажигания

Грузовые автомобили. Система зажигания Сжатая рабочая смесь в цилиндре двигателя зажигается электрическим разрядом – искрой, образующейся между электродами свечи зажигания. Для получения надежного искрового разряда при расстоянии между электродами свечи зажигания 0,5

Контактно – транзисторная система зажигания

Контактно – транзисторная система зажигания В описанной выше системе контактного батарейного зажигания с ростом частоты вращения коленчатого вала двигателя снижается напряжение во вторичной цепи, вызываемое сокращением времени замкнутого состояния контактов

Бесконтактная система зажигания

Бесконтактная система зажигания Контроль за управлением работой транзистора в бесконтактной системе зажигания происходит посредством бесконтактного датчика. Таким датчиком может служить любой преобразователь угла поворота коленчатого вала двигателя в

Неисправности приборов зажигания

Неисправности приборов зажигания Неисправности в системе зажигания приводят к перебоям в работе свечей или полному прекращению искрообразования, а так же к нарушению моментов воспламенения рабочей смеси в цилиндрах.Для проверки наличия тока высокого напряжения

Уход за приборами зажигания

Уход за приборами зажигания Ежедневно проверить внешним осмотром состояние прерывателя – распределителя, свечей зажигания и проводов низкого и высокого напряжения.Первое и второе техническое обслуживание включает в себя: – очистить приборы зажигания внутри от пыли и

Двигатель не запускается в холодное время года

Двигатель не запускается в холодное время года

Не сейчас, Като, продолжай работать венчиком, пока я толкаю MG6 Magnette 1.9 DTi – Tech

Не сейчас, Като, продолжай работать венчиком, пока я толкаю MG6 Magnette 1.9 DTi – Tech Согласно рекламным материалам, MG6 был задуман, спроектирован, реализован и собран в Британии. Это великое британское название и великий британский автомобиль. Кусочек Иерусалима среди темных

При выключении зажигания детонация


Причины детонации двигателя после выключения зажигания

Самопроизвольные вспышки горючей смеси в камерах сгорания — явление непросто неприятное, но и пугающее. На самом деле, если разобраться в причинах этого явления и вовремя предпринять адекватные меры, срок службы двигателя можно продлить. Если научиться отличать детонацию от калильного зажигания, то проблему можно решить ещё проще.

Причина детонации двигателя после выключения зажигания

Самопроизвольные вспышки горючей смеси в камерах сгорания

Процесс, который мы называем детонацией, представляет собой неконтролируемое активное сгорание топлива как во время работы двигателя, так и после его остановки.

Если в нормальном режиме воздушно-топливная смесь может сгорать и распространять фронт пламени со скоростью от 25 до 30 м/с, то во время детонационного процесса фронт распространяется со скоростью в 10–15 раз быстрее. А это уже больше похоже на разрушительный взрыв. Тем не менее детонацию часто путают с калильным зажиганием.

Причина детонации двигателя после выключения зажигания

Калильное зажигание возникает в следствии перегрева деталей камеры сгорания, в основном кокса и нагара на днище поршня, свечах и самой камере. Процесс происходит следующим образом: мы выключаем зажигание, но коленвал по инерции продолжает перемещать поршень вниз, всасывая топливо-воздушную смесь. Она воспламеняется не от искрообразования свечи, а от температуры перегретых деталей. Таким образом, процесс горения может продолжаться ещё несколько секунд, иногда до 10–12.

Калильное зажигание, или всё-таки детонация?

Калильное зажигание

Причинами калильного зажигания могут быть:

  1. В карбюраторных двигателях подача топлива должна перекрываться сразу после выключения зажигания при помощи экономайзера принудительного холостого хода. Именно он может быть причиной нештатной подачи бензовоздушной смеси из-за подклинивания штока клапана. Это происходит потому, что шток может износиться или закоксоваться. Как правило, проблема устраняется чисткой клапана экономайзера принудительного холостого хода, его заменой.
  2. В моторах с инжектором всей системой питания управляет электроника, поэтому причину нужно искать в первую очередь в неисправности датчиков холостого хода, электронном блоке управления двигателем.
  3. В дизельных двигателях причиной калильного зажигания может быть неисправность форсунок, топливного насоса высокого давления, которые также подают солярку в камеру сгорания. В дизелях это чаще может происходит из-за изменения степени сжатия в следствии значительных отложений нагара и в этом случае, действительно, стоит говорить скорее о детонации, чем о калильном зажигании.

Детонация, как она есть

Детонация, как она есть

Детонация может наблюдаться по нескольким причинам с похожими симптомами, но последствия могут быть куда печальнее. При детонации неконтролируемо выделяется огромное количество тепла, катализируя процесс сгорания бензовоздушной смеси.

Основные причины детонации

При схожих симптомах, причинами детонации могут служить:

  1. Использование топлива с низким октановым числом, меньшей детонационной стойкостью, чем рекомендует завод-изготовитель двигателя. В этом случае детонация возникает по причине несоответствия степени сжатия сорту бензина или солярки — они самопроизвольно загораются без искрообразования, из-за высокой степени сжатия и высокой температуры в камере сгорания. Чтобы избежать детонационных процессов, достаточно заправлять автомобиль бензином именно с таким октановым числом, как указывает производитель.
  2. Кроме того, важно и качества топлива, наличие в нём примесей, воды, твёрдых фракций.Слишком раннее зажигание также может вызвать детонацию из-за нарушения температурного режима работы в камере сгорания. В этом случае также рабочая смесь сгорает неконтролируемо, поскольку детали цилиндро-поршневой группы перегреты.
    Несоответствие калильного числа свечей рекомендации завода-изготовителя. В принципе, неправильных свечей не бывает, они просто могут не соответствовать режимам работы конкретного мотора, накаляться и также вызывать детонацию, топливо будет сгорать без искрообразования.
  3. Нагар в камере сгорания, на днище поршня, на клапанах приводит к уменьшению объёма камеры сгорания, а это, в свою очередь, увеличивает степень сжатия. В таком случае, даже на хорошем и соответствующем двигателю бензине может наблюдаться детонация. Мотористы называют несколько причин образования нагара, среди которых эксплуатация двигателя в режимах малых нагрузок. Для профилактики возникновения нагара желательно дать мотору периодически поработать под сильной нагрузкой, на высоких оборотах.
  4. Перегрев двигателя. Если мы уверены в качестве топлива, в состоянии цилиндро-поршневой группы и в правильно установленном угле опережения зажигания, стоит обратить внимания на систему охлаждения: уровне антифриза, корректности работы термостата, а также качестве охлаждающей жидкости, чистоте радиатора охлаждения, работу электрического вентилятора охлаждения радиатора.

Детонация может привести к поломке двигателя

Любая из перечисленных причин, а тем более, их комплекс, могут вызвать детонацию после выключения зажигания, что может привести к полному разрушению двигателя.

Видео об основных причинах появления детонации двигателя

Заключение

Именно поэтому при появлении первых симптомов детонации необходимо проводить диагностику двигателя и принимать меры по устранению неисправности. Чистого всем бензина и ровных дорог!

Почему детонирует двигатель при глушении автомобиля

Исправное состояние мотора характеризуется ровной работой без лишних резких шумов. Любое отклонение от «нормы» не приветствуется – различные стуки и посторонние лязги указывают на критический режим работы деталей. Игнорировать такую симптоматику не рекомендуется – силовая установка может выйти из строя в самый неподходящий момент. Безответственность оценивается не мелкими расходами на диагностику, а крупными затратами на капитальный ремонт.

Что такое детонация и как ее определить

 


Любой автолюбитель может столкнуться с тем, что детонирует двигатель при глушении автомобиля. Но не каждый расценит необычный звук с правильной стороны. Ликвидировав пробелы в вопросе неустойчивых режимов работы мотора, наступит ясность в понятии, допустимо ли это явление или нет.
Определение и суть

Детонация – это процесс горения топливно-воздушной смеси с критически высокой скоростью, приводящий к резкому повышению давления и температуры. Возникает явление на этапе резкого повышения давления в цилиндре и догорания смеси в пристеночных слоях во время такта сжатия.

Мгновенное сгорание подготовленных продуктов вызывает распространение в камере сгорания ударных волн со скоростью до 1 200 м/с. При кондиционном горении также возникают волны ударного характера, однако интенсивность их распространения не превышает 50 м/с.

При столкновении ударной волны с преградами в виде стенок цилиндров и поршней издается характерный детонационный стук. Мнение о том, что это стучат поршневые пальцы, не имеет под собой никакого основания.

Последствия


Чем опасна детонация – логически предположить можно исходя из определения явления. Вполне ясно, что действие ударных волн далеко не лучшим образом сказывается на работоспособности мотора:

  • Повышение отдачи тепловой энергии в днище поршня и стенки камеры сгорания и попутный их перегрев.
  • Разрушение межцилиндровых перегородок и поршней.
  • Ликвидация масляного слоя на стенках цилиндра.
Признаки неисправности

Прежде чем разобраться, из-за чего происходит детонация, необходимо ее выявить. Проявляется нежелательное явление исключительно на работающем моторе. Отсюда следствие – при глушении или после выключения зажигания двигатель детонировать не может. Да и на холостых оборотах встретить ее довольно трудно, разве что при запуске на газу.

А вот под нагрузкой услышать металлические стуки можно. Особенно при разгоне в гору на повышенной передаче и малых оборотах. Ударная волна также противодействует ходу поршня вверх, что выражается в потере мощности и повышенном расходе топлива.

Зеленоватый или черный дым из выхлопной указывает на то, что дело худо. Неприятное явление имело место быть и уже закончилось. Несвоевременная фиксация факта привела к тому, что отколовшиеся части алюминиевых деталей вылетают через выпуск.

 


Основные причины и как их устранить

Стоит проанализировать и недавние изменения, повлекшие за собой возникновение сильных или легких стуков:

  • Посещалась заправка и был залит некачественный или низкооктановый бензин. Руководствуйтесь рейтингом АЗС при выборе автозаправочной сети. В крайнем случае поможет присадка для повышения октанового числа.
  • Система зажигания карбюраторного двигателя подвергалась регулировке. Детонация любит ранее зажигание, поэтому необходимо соблюдать баланс в регулировке угла опережения.
  • «Инжектор» перепрошивался с целью повышения экономичности. Бедная смесь создает благоприятные условия для нестабильной работы.

Детонационный стук может проявляться на холодную или на горячую только при низкой частоте вращения коленчатого вала. На высоких же оборотах он возникает при резком изменении нагрузки или при движении на максимальной скорости.

К сведению. Нагруженные турбодвигатели более подвержены возникновению неустойчивых режимов, нежели атмосферные.

А может ли при глушении двигатель автомашины детонировать: разбираемся в аспектах

Причислять неравномерную работу двигателя или любой другой стук к проявлению детонации ошибочно. Чтобы не ошибаться, лучшим вариантом будет узнать, как звучит детонационный режим на практике. Например, посмотреть тематические видеофайлы.

Дизелинг

Как уже отмечалось, нежелательное явление может появиться исключительно на функционирующем моторе. Как же тогда квалифицировать работу силовой установки при выключенном зажигании? Ответ механиков краток – дизелинг. Природа его иная: самовоспламенение бензина, идентичное рабочему процессу дизельного двигателя.

Наверставшие базу знаний по бензиновому ДВС новички сразу же возразят, приведя пару аргументов «против»: высокооктановое топливо обладает плохой способностью к самостоятельному воспламенению, да и степень сжатия в бензомоторе меньше. Все это верно, но при остановке агрегата создаются благоприятные условия для дизелинга.

 


Исправный двигатель может якобы детонировать при глушении при двух условиях:
  1. Подача топлива в цилиндры.
  2. Низкие обороты коленвала.

На деле процесс выглядит таким образом. Заглушили силовую установку, частота вращения коленчатого вала падает, топливо подается. Время, отведенное на воспламенение смеси, увеличивается.

При таких условиях искры от свечи для поджигания топлива не нужно – достаточно постепенного увеличения давления и температуры. Отработав рабочий такт, обороты коленвала увеличиваются, самовоспламенение не происходит. Далее частота снова падает и дизелинг возникает вновь. И так несколько циклов «дерганья».

Вред или польза

В отличие от стука при качании рулем, ничего опасного в том, что двигатель неустойчиво работает после обесточивания, нет. Наоборот, наличие данного эффекта косвенно подтверждает хорошую герметичность камеры сгорания, что свидетельствует об общей исправности ДВС. Данное явление может происходить только на карбюраторных моторах, потому как на инжекторных силовых установках подача топлива прекращается с выключением зажигания.

Отсюда вывод – отсутствие подергивания после остановки агрегата вовсе не является признаком плохого состояния. К слову, правильно настроенный и ухоженный карбюратор защищает двигатель от появления дизелинга. Реализовано это с помощью электромагнитного клапана системы ЭПХХ, который в исправном состоянии перекрывает подачу горючки в цилиндры при выключении ДВС.

А не калильное ли это зажигание?

Бывалые шоферы часто заменяют понятие дизелинг на калильное зажигание (КЗ), что в корне считается неверным. Элементарные различия раскрывает определение КЗ – это воспламенение топливно-воздушной смеси от нагретого источника, которым может быть:

  • Перегретая поверхность свечи.
  • Выпускной клапан.
  • Нагар.


Как уже определились, двигатель проявляет признаки детонации при глушении от самовоспламенения ТВС при ее сжатии (свечка обесточена). Калильное зажигание подразумевает наличие отклонений именно при работающей свече зажигания: нагретые поверхности или слой нагара воспламеняют смесь раньше, чем необходимо.

Последствия КЗ опасны. Оно может вызвать:

  • Оплавление свечей.
  • Перегрев поршней.
  • Оплавление клапанов.

Примечательно, что «калильные» моторы работают устойчиво во всем диапазоне рабочих оборотов. Устойчивость объясняется тем, что у нагретого источника температура продолжает возрастать и поддерживаться.

Коротко о главном

После остановки двигателя детонации быть не может – это неустойчивое «дерганье» именуется дизелингом. Ничего опасного в себе это явление не несет. Причина его появления – поступление топлива в цилиндры при выключенном зажигании. Встречается, как правило, на карбюраторных двигателях с неисправным ЭМК.

Детонация возникает исключительно на работающем двигателе и сопровождается характерным металлическим звоном. Проявляется при движении на малых оборотах под нагрузкой, при трогании, после заправки низкооктановым бензином и вследствие неправильной установки угла опережения зажигания на карбюраторном моторе. На инжекторной силовой установке за последнее отвечает датчик детонации двигателя и ЭБУ.

Детонация двигателя после выключения зажигания


В старых двигателях этот характерный звук называли «стук пальцев» и отчасти так оно и было. Стучали или звенели, действительно, поршневые пальцы, но сегодня в корне изменились материалы и точность их обработки, поэтому возникающий стук пальцев уже не такое безобидное явление, как было 20 лет назад. Детонация. Почему она возникает и как с ней бороться, разберемся вместе.

Содержание:

  1. Что такое детонация
  2. Детонация при выключенном зажигании
  3. Степень сжатия и нагар в камере сгорания
  4. Угол опережения зажигания и конструкционные недостатки

Что такое детонация

В нормально работающем двигателе топливо сгорает только по команде. Никакой самодеятельности. Двигатель всегда должен быть под контролем. Иначе это грозит серьезными проблемами. «Мотор пошел вразнос», очень неприятное явление, когда двигатель продолжает набирать обороты даже после выключения зажигания, не реагируя ни на дроссельную заслонку, ни на подачу искры. Это крайний случай, к которому может привести детонация, а в зародыше мы ее слышим, как стук пальцев.

Этот звук, не что иное, как взрывная волна, которая отражается от стенок цилиндров. А в принципе, детонация это превышение скорости пламени сгорания топливо-воздушной смеси в сотни раз. В нормальном режиме работы пламя имеет скорость около 15 м/с, а при детонационных процессах — 2500 м/с. Можно только представить себе, какая нагрузка ложится на детали кривошипно-шатунного механизма, на прокладки, на стенки камеры сгорания и выхлопную систему.

Повышенные вибрации, перегрев, падение мощности — это только некоторые последствия детонации, а приводят к ее появлению слишком раннее зажигание, низкооктановое топливо, перегрев мотора, неправильные регулировки фаз газораспределения и прочие неисправности.

Детонация при выключенном зажигании

 

Детонация может возникнуть и при выключенном зажигании. Это проявляется в том, что двигатель продолжает нестабильно работать после попытки его заглушить секунд 15-20. Наблюдаются вспышки в цилиндрах, которые контролировать нет никакой возможности. Причин детонации может быть множество, но мы остановимся на самых распространенных и основных:

  • перегрев двигателя;
  • высокая степень сжатия;
  • нагар в камере сгорания;
  • применение низкооктанового топлива;
  • слишком большой угол опережения зажигания;
  • несоответствие калильного числа свечей условиям работы двигателя.

Теперь рассмотрим более подробно каждую из причин. При перегреве двигателя детонация возникает вследствие очень высокой температуры в камере сгорания. Настолько высокой, что для воспламенения топливу уже не нужна искра. Ему достаточно просто попасть в камеру сгорания и происходит неконтролируемый взрыв. Частота взрывов может привести к полному разрушению кривошипно-шатунного механизма и головки блока цилиндров.

Степень сжатия и нагар в камере сгорания

Если топливо, которое применяется для работы двигателя, не соответствует степени сжатия в камере сгорания, детонация при нагреве мотора неизбежна. К примеру, если двигатель рассчитан на бензин А 92, а вместо него в бак попал А 76, можно ждать детонации в ближайшее время. Октановое число топлива, которое заявлено производителем, должно обеспечивать работу двигателя при определенном давлении. Если степень сжатия выше, чем допустимая для конкретной марки бензина, то он может сгорать самопроизвольно, что и приводит к детонации.

Вместе с тем, нагар, который образуется в камере сгорания в процессе длительной работы двигателя, уменьшает объем камеры сгорания, тем самым увеличивая степень сжатия в цилиндре. В этом случае двигатель может детонировать даже на нормальном бензине. Еще одна опасность, которую приносит нагар, это ухудшение теплообмена в камере сгорания, что приводит к очень быстрому нагреву деталей: поршня, поршневых колец, клапанов, самой головки блока цилиндров. Нередко именно нагар и стает причиной, по которой возникает детонация двигателя после выключения зажигания.

Угол опережения зажигания и конструкционные недостатки

При чем здесь зажигание? Очень просто. Раннее зажигание приводит к тому, что пиковое значение давления в камере сгорания сдвигается ближе к верхней мертвой точке. Как мы уже выяснили, чем выше давление в процессе сгорания смеси, тем выше вероятность возникновения детонации.

Плюс некачественный бензин и нагар, в результате этот комплекс неприятностей приводит к детонации. Есть также ряд чисто конструкционных недостатков, влияющих на возникновение детонации:

  1. Недостаточный уровень охлаждения двигателя. Следует убедиться, что система охлаждения исправна, а радиатор нормально обдувается встречным воздухом, термостат в рабочем состоянии и уровень антифриза в норме.
  2.  Замедленный процесс догорания топлива из-за неправильно подобранного калильного числа свечей. Здесь замена свечей поможет решить вопрос положительно.
  3. Неудачная конструкция камеры сгорания, препятствующая нормальной вентиляции и отводу тепла.

Детонацию можно побороть, только комплексно изучив все нюансы, связанные с перегревом двигателя, но те моменты, на которые мы указали стоит проверить особо тщательно. Не перегревайте мотор, и прохладной вам дороги!

Читайте также Гидроудар двигателя — что это такое

Читайте также:


причины, как исправить, чем грозит

Детонация в двигателе после выключения мотора — крайне неприятная проблема, с которой может столкнуться любой водитель. Причиной детонации является самовозгорание топлива после завершения работы мотора, а последствия от такой проблемы могут быть самые разные, вплоть до повреждения ключевых элементов двигателя и необходимости проведения капитального ремонта. В рамках данной статьи рассмотрим, что делать, если после выключения зажигания происходит детонация.


Оглавление: 
 1. Чем грозит самопроизвольная детонация
 2. Как диагностировать самопроизвольную детонацию
 3. Почему возникает самопроизвольная детонация после выключения двигателя
 4. Что делать при самопроизвольной детонации
 

Чем грозит самопроизвольная детонация

Перед тем как разбираться с причинами проблемы, важно более подробно ознакомиться с ситуацией. Самопроизвольная детонация — это крайне нежелательный процесс для любого двигателя. При возгорании и взрыве топлива вне времени работы мотора происходит мощное физическое воздействие на всю цилиндро-поршневую группу. Соответственно, в результате такой детонации наносится вред цилиндрам, поршням, шатунам, коленвалу и другим деталям.

Важно: Нагрузки в момент самопроизвольной детонации после выключения мотора достаточно высокие, и со временем они навредят работе мотора в любом случае, поэтому важно выявить и устранить проблему на ранних этапах.

Как диагностировать самопроизвольную детонацию

Определить, что после выключения зажигания в двигателе происходит детонация, очень просто. На это указывает соответствующий звук.

Обратите внимание: Самопроизвольная детонация после выключения двигателя — это не моментальная реакция. Процесс длится на протяжении 20-30 секунд, в течение которых слышны хлопки со стороны двигателя.

Многие неопытные водители считают, что подобная ситуация с самопроизвольной детонацией является нормальной. На самом деле это не так. После выключения зажигания никаких посторонних двигателей от мотора не должно исходить.

Важно: Если вы диагностировали, что у вас после выключения двигателя автомобиля происходит в нем самопроизвольная детонация, не следует затягивать с решением проблемы. Обязательно самостоятельно установите причину такой неисправности устраните ее, либо обратитесь в сервисный центр. Если вовремя не предпринять шаги для устранения самопроизвольной детонации после выключения двигателя, это может привести к серьезным проблемам в работе мотора.

Почему возникает самопроизвольная детонация после выключения двигателя

Можно выделить две основные причины возникновения самодетонации после выключения зажигания:

  • Использование топлива, которое не предназначено для данного автомобиля. Речь идет, например, о бензине с более низким октановым числом, чем рекомендует производитель. Бензин с низким октановым числом предполагается использовать на моторах, степень сжатия в которых невысока. Если, например, для двигателя рекомендуется бензин АИ-95, а в него налить топливо АИ-80, то оно будет самопроизвольно детонировать;
  • Неправильно выставленное зажигание. Чтобы топливо детонировало, зажигание выставляется ранним. Это приводит к тому, что возгорание бензина происходит в момент движения поршня на сжатие. Соответственно, возникает перегрев рабочего пространства двигателя. Из-за повышенной температуры детонации происходят непроизвольно и после выключения зажигания.

Это наиболее частые причины, почему возникает самопроизвольная детонация топлива. Но также нельзя исключать вариант, что проблема кроется в свечах. Если используются свечи, которые не рекомендуют производителем, может возникать такая проблема.

Что делать при самопроизвольной детонации

Если вы замечаете, что после выключения двигателя у вас в моторе происходит самопроизвольная детонация, рекомендуется:

  1. Убедиться по технической литературе к автомобилю, что используется рекомендованное производителем топливо. Возможно, потребуется сменить заправку на более вызывающую доверие, способную предоставить топливо лучшего качества;
  2. Далее проверьте, как выставлено зажигание. Если установлено раннее зажигание, настройте его ближе к средним значениям;
  3. Проверьте свечи зажигания, обратите внимание на нагар на них. Используйте только свечи, которые рекомендует производитель.

Самопроизвольная детонация — крайне неприятное явление, которое следует устранить сразу после обнаружения первых симптомов.

Загрузка…

Детонация двигателя при выключении зажигания

Детонацией называется горение смесей в цилиндрах мотора. Определить данную проблему можно по громкому металлическому звуку. Последствия могут быть самыми плачевными, вплоть до поломки. Часто возникает во время работы мотора при повышенных оборотах (подъём автотранспорта в гору и так далее). Однако также случается детонация двигателя при выключении зажигания.

Основные причины детонации при выключении зажигания:

  • неподходящее топливо: если топливо не соответствует типу, необходимому для нормальной работы ДВС, происходит активное сгорание, близкое к скорости взрыва. Это случается, когда автомобиль вместо положенного АИ-95, заправляют АИ-92. Данное явление служит причиной выделения избытков тепловой энергии, которая отрицательно воздействует на работу двигателя;

Следите за качеством бензина. Заправляйтесь только на проверенных автозаправках.

  • другая причина, по которой двигатель детонирует после выключения зажигания, – чрезмерно раннее зажигание. Необходимость данной установки возникает, для повышения восприимчивости мотора к отворению дроссельной заслонки, что в свою очередь может привести к избыточному нагреванию. Ещё одна причина раннего зажигания – не подходящие свечи к данному типу двигателя. Свечи держат под контролем горючую смесь, не давая ей сильнее воспламеняться. При высокой степени горения, изменяется объём камеры сгорания, затем повышается температура;
  • перегрев – ещё одно возможное основание, чтобы понять, почему детонирует двигатель. Для этого необходимо проверить количество охлаждающей жидкости и термостат.

Длительное использование автомобиля на низких скоростях может стать причиной появления нагара. ДВС время от времени необходимы большие нагрузки при соблюдении правил дорожного движения.

Детонация внутри двигателя

Помните, что своевременное техническое обслуживание и проверка автомобиля сразу при возникновении внештатных ситуаций, сбережёт Ваш автомобиль и позволит избежать дополнительных трат и аварийных ситуаций на дороге.

Facebook

Twitter

Вконтакте

Google+

Понравилась статья? Поделиться с друзьями:

Детонация при выключении зажигания | Audi Club Russia

Машина: Audi 80 B3, год выпуска 1988, двигатель 1,8 PM(моновпрыск).

Проблема: Заводиться хорошо, едет хорошо, и все было бы отлично, но есть одно но… Выключаю зажигание и машину начинает трясти — детонация. В чем может быть дело не знаю. Поменял свечи, поставил NGK – нагара на них не образуется, проверял, да и старых нагара было уж не так и много. Залил присадку Liqui moly для чистки клапанов, так как сказали, что может из-за нагара калильное зажигание, не помогло L Бензин заливаю 92, на проверенной заправке (проверенной др. людьми, пробовал и на других разницы для меня ни какой — детонирует). Недавно поменял масло и фильтры (воздушный и масляный).

*******:namespace prefix = o ns = «urn:schemas-microsoft-comfficeffice» /><o></o>

Сам я чайник – это первая машина….но хочу во всем разбираться сам!!! ПОМОГИТЕ может кто сталкивался с такой проблемой.

 

Детонация и предварительное зажигание

ЧТО ТАКОЕ ДЕТОНАЦИЯ?

Детонация (также называемая «искровым детонацией») — это случайная форма горения, которое может вызвать выход из строя прокладки головки, а также другие повреждения двигателя. Детонация возникает при чрезмерном нагревании и давлении в камере сгорания. вызвать самовоспламенение топливно-воздушной смеси. Это создает несколько фронтов пламени. внутри камеры сгорания вместо одного ядра пламени. Когда эти несколько пламен сталкиваются, они делают это со взрывной силой, которая вызывает внезапное повышение давления в баллоне, сопровождающееся резким металлическим звоном или стуком шум.Ударные волны, похожие на молот, создаваемые детонацией, поражают голову прокладку, поршень, кольца, свечи зажигания и шатунные подшипники до сильных перегрузок.

Небольшая или случайная детонация может произойти практически в любом двигателе и обычно не причиняет вреда. Но длительная или сильная детонация может быть очень разрушительной. Поэтому, если вы слышите стук или звон при ускорении или буксировке двигателя, у вас наверняка проблема с детонацией.

ДЕСЯТЬ СПОСОБОВ ПРЕДОТВРАЩЕНИЯ ДЕТОНАЦИИ

1.Попробуйте использовать топливо с более высоким октановым числом. Октановое число данного сорта бензин — это показатель его детонационной стойкости. Чем выше октан число, тем лучше топливо сопротивляется детонации. Большинство двигателей в хорошем состоянии состояние будет работать нормально на обычном топливе с октановым числом 87. Но двигатели с высоким степеней сжатия (более 9: 1), турбокомпрессоров, нагнетателей или с накоплением Отложения углерода в камере сгорания могут потребовать топлива с октановым числом 89 или выше.
Способ использования транспортного средства также может повлиять на его октановые требования.Если автомобиль используется для буксировки или другого применения, когда двигатель вынужден работать При работе под нагрузкой может потребоваться топливо с более высоким октановым числом для предотвращения детонации.

Если при переходе на топливо с более высоким октановым числом не удается устранить постоянная проблема детонации, вероятно, это означает, что что-то не так. Все, что увеличивает нормальную температуру или давление сгорания, выходит наружу. воздушно-топливной смеси или заставляет двигатель работать более горячим, чем обычно, может вызвать детонация.

2. Проверьте систему рециркуляции ОГ. Система рециркуляции отработавших газов (EGR) Система является одним из основных средств контроля выбросов двигателя. Его цель — уменьшить загрязнение выхлопных газов оксидами азота (NOX). Это происходит за счет «утечки» (рециркуляция) небольшого количества выхлопных газов во впускной коллектор через Клапан рециркуляции ОГ. Хотя газы горячие, на самом деле они оказывают охлаждающее действие на температуры сгорания путем небольшого разбавления топливовоздушной смеси. Снижение температура сгорания снижает образование NOX, а также октановое число требования двигателя.
Если клапан рециркуляции ОГ не открывается, потому что сам клапан неисправен или его подача вакуума заблокирована (ослабленные, забитые или неправильно проложенные соединения вакуумного шланга или регулирующий клапан или соленоид), охлаждающий эффект теряется. Результат будет более высокая температура сгорания под нагрузкой и повышенная вероятность детонации.

Конфигурацию и прокладку шлангов см. В руководстве по обслуживанию. системы рециркуляции отработавших газов вашего двигателя и рекомендуемую процедуру проверки работа системы EGR.

3. Сохраняйте сжатие в разумных пределах. Статическое сжатие соотношение 9: 1 обычно является рекомендуемым пределом для большинства безнаддувных уличные двигатели (хотя некоторые более новые двигатели с датчиками детонации могут работать с более высокими степени сжатия).
Степень сжатия более 10,5: 1 может создать проблема детонации даже с бензином премиум-класса с октановым числом 93. Так что если двигатель не построены для работы на гоночном топливе, поэтому степень сжатия должна быть в пределах разумный пробег по насосу бензина.Это, в свою очередь, может потребовать использования более низких поршни сжатия и / или головки цилиндров с камерами сгорания большего размера. Другой вариант — использовать прокладку под прокладку из меди с головкой приклада. прокладка для уменьшения компрессии.

Задержка фаз газораспределения может также снизить давление в цилиндре до уменьшить детонацию на низких оборотах, но это повредит крутящий момент на низкой скорости, который не рекомендуется для уличных двигателей или автомобилей с автоматикой.

Для применений с наддувом или турбонаддувом, статическое сжатие соотношение 8: 1 или меньше может потребоваться в зависимости от величины давления наддува.

Еще один момент, о котором следует помнить, — растачивание цилиндров двигателя возможность установки поршней увеличенного размера также увеличивает степень статического сжатия. Так же делает фрезеровку ГБЦ. Если такие модификации необходимы компенсировать износ цилиндра, коробление головки или повреждение, возможно, вам придется использовать более толстая прокладка головки, если она доступна для применения, или прокладка головки прокладка (мертвая мягкая медная прокладка), чтобы компенсировать увеличение сжатия.

4.Проверьте установку опережения зажигания. Слишком много искры может вызвать слишком быстрое повышение давления в цилиндре. Если сбросить время на стоковые характеристики не помогают, задержка времени на пару градусов и / или повторная калибровка кривой продвижения дистрибьютора может потребоваться для сохранения детонация под контролем.

5. Проверьте датчик детонации на предмет неисправности. Многие двигатели поздних моделей есть «датчик детонации» на двигателе, который реагирует на частоту вибрации, характерные для детонации (обычно 6-8 кГц).В Датчик детонации выдает сигнал напряжения, который сигнализирует компьютеру о том, что замедлить момент зажигания до прекращения детонации.
Если флажок » «двигатель» горит, проверьте бортовую компьютерную систему автомобиля с помощью предписанная процедура для «кода неисправности», который соответствует неисправный датчик детонации (код 42 или 43 для GM, код 25 для Ford или код 17 для Крайслер).

Датчик детонации обычно можно проверить, постучав гаечным ключом по коллектор рядом с датчиком (никогда не ударяйте по датчику!) и следите за изменение времени при работе двигателя на холостом ходу.Если отсчет времени не замедляется, датчик может быть неисправен или проблема может быть в электронной искре схема управления синхронизацией самого компьютера. Чтобы определить причину, вы необходимо обратиться к соответствующей диагностической таблице в руководстве по обслуживанию и следовать пошаговые процедуры тестирования для выявления причины.
Иногда стук датчик будет реагировать на звуки, отличные от звуков детонации. Шумный механический топливный насос, неисправный водяной насос, подшипник генератора или ослабленный шток подшипник может производить вибрации, которые могут обмануть датчик детонации и заставить его сроки.

6. «Прочтите» свечи зажигания. Взять их заменено, если необходимо. Неправильная вилка диапазона нагрева может вызвать детонацию, а также преждевременное воспламенение. Если изоляторы вокруг электроды на ваших вилках кажутся желтоватыми или покрытыми пузырями, они могут быть слишком горячими для приложение. Попробуйте следующий диапазон температур холоднее свеча зажигания. Искра с медным сердечником свечи обычно имеют более широкий диапазон нагрева, чем обычные заглушки, что уменьшает опасность взрыва.

7.Проверить двигатель на перегрев. Горячий двигатель с большей вероятностью получить искровую детонацию, чем та, которая работает при нормальной температуре. Может перегрев быть вызвано низким уровнем охлаждающей жидкости, проскальзыванием муфты вентилятора, слишком маленьким вентилятором горячий термостат, неисправный водяной насос или даже отсутствующий кожух вентилятора. Плохая жара теплопроводность в головке и водяной рубашке может быть вызвана отложением извести отложения или паровые карманы (которые могут возникнуть из-за застрявших воздушных карманов).

8. Проверить работу системы забора нагретого воздуха.В Работа воздухоочистителя с термостатическим управлением заключается в создании карбюраторного двигателя. горячим воздухом при холодном пуске двигателя. Это способствует испарению топлива. во время прогрева двигателя. Если дверца воздушной заслонки закрывается или открывается медленно чтобы карбюратор продолжал получать нагретый воздух после прогрева двигателя, добавленного тепла может быть достаточно, чтобы вызвать проблему детонации — особенно во время жаркая погода. Проверить работу заслонки управления воздушным потоком в воздухе очистите, чтобы убедиться, что он открывается при прогреве двигателя.Отсутствие движения может означать вакуумный двигатель или термостат неисправен. Кроме того, проверьте клапан стояка тепла на убедитесь, что он открывается правильно, так как это тоже может повлиять на систему забора воздуха.

9. Проверьте обедненную топливную смесь. Богатые топливные смеси сопротивляются детонация, а тощие — нет. Утечки воздуха в вакуумных магистралях, впускном коллекторе прокладки, прокладки карбюратора или впускной трубопровод после топлива дроссельная заслонка впрыска может впускать дополнительный воздух в двигатель и откачивать топливо смесь.Бедная смесь также может быть вызвана загрязнением топливных форсунок, карбюратора. жиклеры засорены отложениями топлива или грязью, засорение топливного фильтра или слабое топливо насос.
Если топливная смесь становится слишком бедной, возможны пропуски зажигания. возникают при увеличении нагрузки на двигатель. Это может вызвать колебание, споткнуться и / или проблема грубого холостого хода.
На соотношение воздух / топливо также может влиять по изменению высоты. По мере того, как вы поднимаетесь вверх, воздух становится менее плотным.
Карбюратор, откалиброванный для езды на большой высоте, будет работать слишком бедно, если проехали на более низкой отметке.Изменение высоты обычно не проблема двигатели с карбюраторами с электронной обратной связью или электронным впрыском топлива потому что датчики кислорода и барометрического давления компенсируют изменения в воздухе плотность и соотношение топлива.

10. Удалите нагар. Накопление углеродных отложений в в камере сгорания и в верхней части поршней может увеличивать сжатие до точка, где детонация становится проблемой. Отложения углерода — обычное является причиной детонации в двигателях с большим пробегом, и может быть особенно толстым, если двигатель потребляет масло из-за износа направляющих и уплотнений клапанов, износа или поломки износ поршневых колец и / или цилиндров.Нечастая езда и не замена масла достаточно часто может также ускорить накопление депозитов.
В дополнение к увеличивая сжатие, нагар также имеет изолирующий эффект, который замедляет нормальный перенос тепла из камеры сгорания в голова. Поэтому толстый слой отложений может повысить температуру горения и способствуют «преждевременному зажиганию», а также детонации.
Углерод отложения часто можно удалить с двигателя, который все еще находится в эксплуатации, с помощью химический «верхний очиститель».»Этот тип продукта заливается в холостой ход двигатель через карбюратор или дроссельную заслонку. Затем двигатель выключается, поэтому растворитель может впитаться и ослабить отложения. При перезапуске двигателя отложения выдуваются из камеры сгорания.
Если химическая очистка не удается удалить отложения, может потребоваться вытащить головку блока цилиндров и соскребите отложения металлической щеткой или скребком (будьте осторожны, чтобы не поцарапать лицевой стороной головки блока цилиндров или моторного отсека!).

11.Проверить давление наддува. Контроль количества наддува в Двигатель с турбонаддувом абсолютно необходим для предотвращения детонации. Турбо перепускная заслонка сбрасывает давление наддува в ответ на подъем впускного коллектора давление. На большинстве двигателей последних моделей помогает электромагнитный клапан с компьютерным управлением. регулируют работу вестгейта. Неисправность коллектора датчик давления, соленоид управления перепускным клапаном, сам перепускной клапан или утечка в вакуумных соединениях между этими компонентами может позволить турбо доставляет слишком много наддува, что разрушает прокладку головки, а также двигатель в короткий заказ, если не исправить.
Улучшенное промежуточное охлаждение может помочь снизить детонация при наддуве. Работа интеркулера — понижать уровень поступающего воздуха. температура после выхода из турбокомпрессора. Добавление интеркулера в турбомотор без промежуточного охлаждения (или установка большего или более эффективного интеркулер) может устранить проблемы детонации, а также позволить двигателю безопасно обрабатывать больше наддува.

12. Измените привычки вождения. Вместо того, чтобы тащить двигатель, попробуйте переключение на более низкую передачу и / или более плавное ускорение.Иметь ввиду, Кроме того, двигатель и трансмиссия должны соответствовать условиям эксплуатации. Если вы слишком много работаете с двигателем, возможно, вам нужна коробка передач с более широкое передаточное число или более высокое передаточное число главной передачи в дифференциале.

ПРЕДВАРИТЕЛЬНОЕ ЗАЖИГАНИЕ

Еще одно состояние, которое иногда путают с детонацией, — это «предварительное зажигание». Это происходит, когда точка внутри камеры сгорания становится настолько горячей, что становится источником воспламенения и вызывает воспламенение топлива перед свечой зажигания. пожары.Это, в свою очередь, может способствовать или вызвать проблему детонации.

Вместо зажигания топлива в нужный момент дать коленвал плавный толчок в нужную сторону, топливо преждевременно воспламеняется (рано) вызывает кратковременный люфт, когда поршень пытается повернуть шатун неправильное направление. Это может быть очень опасно из-за создает. Он также может локализовать тепло до такой степени, что может частично расплавить или прожечь верхнюю часть поршня!

Предварительное зажигание может также дать о себе знать, когда горячий двигатель выключен выкл.Двигатель может продолжать работать даже при включенном зажигании. выключен, поскольку камера сгорания достаточно горячая для самовозгорания. В двигатель может продолжать работать или «дизель» и хаотично пыхтеть в течение несколько минут.

Чтобы этого не произошло, в некоторых двигателях есть «топливо». соленоид отключения »на карбюраторе, чтобы остановить подачу топлива в двигатель после выключения зажигания. Другие используют «соленоид остановки холостого хода». который полностью закрывает дроссельную заслонку, чтобы перекрыть подачу воздуха в двигатель.Если любое из этих устройств неправильно настроено или не работает, может возникнуть проблема с выбегом. Двигатели с электронным впрыском топлива не имеют этой проблемы, потому что форсунки прекращают разбрызгивание топлива, как только выключается зажигание.

ПРИЧИНЫ ПРЕДВАРИТЕЛЬНОГО ЗАЖИГАНИЯ

Углеродистые отложения образуют тепловой барьер и могут способствовать фактор предзажигания. К другим причинам относятся: перегретая свеча зажигания (тоже горячий диапазон нагрева для применения). Светящийся нагар на горячем выхлопе клапан (что может означать, что клапан слишком горячий из-за плохой посадки, слабая пружина клапана или недостаточный зазор клапана).

Острый край в камере сгорания или на верхней части поршня (Закругление острых кромок болгаркой может устранить эту причину).

Острые кромки на неправильно переточенных клапанах (недостаточно маржа слева по краям).

Бедная топливная смесь.

Низкий уровень охлаждающей жидкости, пробуксовка муфты вентилятора, не работает электрическая вентилятор охлаждения или другая проблема в системе охлаждения, из-за которой двигатель работает сильнее чем обычно.




Напишите мне на [email protected]

Вернуться в дом брата Боба Страница

Вернуться на главную страницу (верхний уровень)

Авторские права © 1997 Боб Хьюитт — Все права защищены

.

Детонация

Детонация (также называемая «искровым детонацией») — это неустойчивая форма сгорания, которая возникает, когда в камерах сгорания двигателя одновременно возникают несколько фронтов пламени. Вместо единого фронта пламени, расширяющегося наружу от точки воспламенения, в камере сгорания самопроизвольно возникают множественные фронты пламени. Когда несколько фронтов пламени сталкиваются, они производят резкий металлический звон или стук, который предупреждает вас о том, что происходят неприятные вещи.

Если в вашем двигателе есть проблема с детонацией, вы скорее всего услышите ее при ускорении под нагрузкой, при подаче газа в двигатель, когда вы находитесь на высокой передаче или когда тащите двигатель. Детонация возникает из-за того, что топливо с октановым числом (мера его сопротивления детонации) не выдерживает повышенного тепла и давления, когда двигатель находится под нагрузкой. Когда это происходит, топливная смесь самовоспламеняется, создавая разрушительные многочисленные фронты пламени.

Легкая детонация может произойти практически в любом двигателе и не причинит никакого вреда.Но продолжительная сильная детонация — плохая новость, потому что она забивает поршни и кольца. Если проблему не устранить, сильная детонация может повредить ваш двигатель. Это может привести к растрескиванию поршней и колец, разрушению прокладки головки, повреждению свечей зажигания и клапанов и даже к сплющиванию подшипников штока.

Детонация также приводит к потере мощности, так как повышение давления в цилиндре происходит слишком быстро для эффективного рабочего хода. Вместо того, чтобы расти постепенно, он слишком быстро достигает пика, а затем спадает.Результат больше похож на внезапный удар, чем на сильный, устойчивый толчок.


ПРЕДОТВРАЩЕНИЕ ДЕТОНАЦИИ БЕНЗИНОМ С ВЫСОКИМ ОКТАНОМ

Один из способов предотвратить детонацию — использовать топливо с более высоким октановым числом. Октановое число моторного топлива является мерой его сопротивления детонации. Октановое число, указанное на насосе заправочной станции, называется «октановым числом насоса», которое является средним октановым числом исследовательских и моторных. Метод определения октанового числа топлива варьируется в зависимости от используемого метода, но чем выше октановое число, тем лучше топливо сопротивляется детонации.Топливо с октановым числом 87 менее устойчиво к детонации, чем топливо с рейтингом 89 или 91.

Октановое число бензина может быть улучшено за счет дополнительной очистки для увеличения доли более тяжелых углеводородов в топливе, за счет использования сырой нефти более высокого качества или путем добавления этанолового спирта в качестве усилителя октанового числа (все это может увеличить стоимость топлива) .

Тетраэтилсвинец долгое время использовался в качестве антидетонационной присадки для повышения октанового числа бензина. Это была самая эффективная и наименее дорогая добавка, которую можно было использовать для этой цели.Но длительное воздействие свинца связано с многочисленными рисками для здоровья. Этилированный бензин был выведен из употребления в США еще в 1970-х годах, поэтому для повышения его эффективности используется усиленная переработка (крекинг, изомеризация и другие процессы). октановое число базового бензина. Добавлены дополнительные усилители октанового числа, такие как МБТЭ, этанол, ароматические углеводороды и сильно разветвленные алканы. к бензину, чтобы соответствовать требованиям к октановому числу для адекватного сопротивления детонации.

ПОСЛЕПРОДАЖНЫЕ ДОБАВКИ ДЛЯ ПОВЫШЕНИЯ ОКТАНА

Если вы управляете старым маслкаром и не можете найти бензин с достаточно высоким октановым числом, чтобы предотвратить детонацию в вашем двигателе, и вы не хотите расстраивать двигатель, замедляя синхронизацию зажигания или уменьшая его степень сжатия, вы можете добавить добавка для повышения октанового числа бензина в топливный бак.Некоторые присадки, повышающие октановое число, также содержат свинец или заменители свинца для защиты выпускных клапанов в двигателях до 1973 года (в которых отсутствуют упрочненные седла клапанов) от преждевременного износа. Такие продукты могут повысить октановое число перекачиваемого газа на несколько пунктов в зависимости от используемой концентрации (всегда следуйте инструкциям). Но даже этого может быть недостаточно, чтобы устранить постоянную проблему детонации искры, если степень сжатия вашего двигателя превышает 10: 1, или он имеет наддув или турбонаддув.


ЧТО ВЫЗЫВАЕТ ДЕТОНАЦИЮ?

Детонация может иметь несколько причин.Все, что увеличивает температуру или давление сгорания (например, турбонаддув или наддув), или увеличивает рабочую температуру двигателя, увеличивает риск детонации. Повышенная синхронизация зажигания или что-либо, что приводит к тому, что топливно-воздушная смесь работает более бедной, чем обычно, также может вызвать детонацию.

Для некоторых двигателей требуется топливо высшего качества (с октановым числом 91 или выше), и при заправке топливного бака средним или обычным топливом может возникнуть детонация. При небольшом открытии дроссельной заслонки двигатель может нормально работать на менее дорогом топливе, но при резком ускорении или при буксировке двигателя под нагрузкой может произойти детонация.

Предполагается, что датчик детонации обнаруживает вибрации, которые сигнализируют о детонации, и временно замедляет синхронизацию зажигания, пока детонация не прекратится. Но даже в этом случае он не может полностью предотвратить детонацию. Мы советуем использовать сорт бензина, рекомендованный в руководстве по эксплуатации или напечатанный на крышке топливного бака, чтобы минимизировать риск детонации.

Другие причины детонации могут включать любую из следующих:

Слишком сильное сжатие может вызвать детонацию. Накопление нагара в камерах сгорания, на крышках поршней и клапанах может увеличить сжатие до точки, где это вызовет детонацию. Отложения углерода также могут вызвать «преждевременное зажигание», то есть состояние, при котором горячие точки в камере сгорания становятся точками воспламенения, в результате чего топливо воспламеняется до возгорания свечи зажигания. Предварительное зажигание также заставляет двигатель работать после выключения зажигания.

Скорость накопления отложений зависит от типа вождения и качества сжигаемого топлива.Отложения углерода постепенно накапливаются в новом двигателе в течение первых 5000-15000 миль, а затем выравниваются. Состояние равновесия достигается, когда старые отложения отслаиваются примерно с той же скоростью, что и новые отложения. Нечастое вождение, нечастая замена масла или внутренние проблемы двигателя, такие как изношенные направляющие клапана, или изношенные, сломанные или неправильно установленные кольца, которые допускают горение масла, могут значительно ускорить накопление отложений.

Чтобы избавиться от отложений, вылейте баллончик со средством для чистки верха в карбюратор или через корпус дроссельной заслонки, когда двигатель работает на холостом ходу (следуйте инструкциям на продукте).Дайте химическому веществу впитаться в течение рекомендованного периода времени, затем перезапустите двигатель и продуйте грязь (после этого рекомендуется заменить масло). При необходимости повторите, если первая очистка не устранила проблему детонации.

Если химическая очистка не удаляет нагар, всегда можно использовать метод «Italian Tuneup» для удаления нагара из двигателя. Отведите свой автомобиль в место, где мало или совсем нет движения, и вы можете безопасно разогнаться на полном газу до указанного ограничения скорости (или выше, если вы не против рисковать штрафом за превышение скорости).Повторите это несколько раз, затем продолжайте движение на скоростной автомагистрали не менее 15 минут, чтобы удалить нагар из камер сгорания.

Если двигатель с большим пробегом настолько сильно нагревается, что химическая очистка и / или жесткое вождение не могут удалить нагар, другой вариант — использовать «мягкие» абразивные среды, такие как измельченные скорлупы грецких орехов, для очистки камер сгорания. Эту работу можно выполнить с головкой блока цилиндров на месте, сняв свечу зажигания, выдув носитель через свечное отверстие, чтобы выбить нагар, а затем высасывая мусор с помощью заводского вакуума.

Если у вашего двигателя степень статического сжатия выше 10: 1, единственный способ полностью устранить проблему детонации в насосе газа может быть перестроить двигатель с поршнями с более низким сжатием или головками цилиндров с большими камерами сгорания, или замените стоковую прокладку головки на более толстую, чтобы снизить степень сжатия!

Чрезмерная установка угла опережения зажигания может вызвать детонацию . Слишком большое опережение искры приводит к слишком быстрому росту давления в цилиндре.На старых автомобилях с механическим распределителем вращение распределителя для замедления синхронизации на несколько градусов и / или замена пружин опережения зажигания, чтобы синхронизация не двигалась так быстро, может снизить риск детонации, но это также ухудшит производительность. На более новых автомобилях с электронной системой синхронизации зажигания можно изменить кривую опережения зажигания с помощью специального диагностического прибора тюнера.

Перегрев двигателя может вызвать детонацию . В горячем двигателе больше шансов получить искровую детонацию, чем в двигателе, работающем при нормальной температуре.Перегрев может быть вызван низким уровнем охлаждающей жидкости (проверьте на наличие утечек), неисправной муфтой вентилятора, недостаточным размером вентилятора или отсутствующим кожухом вентилятора, электрическим вентилятором системы охлаждения, реле вентилятора или датчиком температуры, которые не работают должным образом, термостатом, который заедает закрыто, неисправный водяной насос, забитый радиатор или серьезное ограничение в выхлопе, такое как засоренный каталитический нейтрализатор, отводящий тепло в двигатель. Плохая теплопроводность внутри двигателя из-за скопления ржавчины или накипи внутри охлаждающих рубашек двигателя также может привести к перегреву двигателя.Проверьте работу охлаждающего вентилятора (электрические вентиляторы должны включаться при включении кондиционера) и проверьте на утечки охлаждающей жидкости. Проверить состояние охлаждающей жидкости. В случае загрязнения добавьте бутылку очистителя системы охлаждения в систему охлаждения, дайте ей поработать в течение указанного периода времени, затем слейте воду и промойте систему охлаждения.

Перегретый воздух может вызвать детонацию . На старых автомобилях с карбюраторами воздухоочиститель с термостатическим управлением подает горячий воздух, чтобы способствовать испарению топлива во время прогрева двигателя.Если дверца воздушной заслонки закрывается так, что карбюратор продолжает получать нагретый воздух после прогрева двигателя, двигатель может взорваться, особенно в жаркую погоду. Проверьте работу заслонки управления потоком воздуха в воздухоочистителе, чтобы убедиться, что она открывается при прогреве двигателя. Отсутствие движения может означать, что вакуумный двигатель или термостат неисправны.

Если у вас есть воздухоочиститель открытого типа на более старом двигателе с карбюратором или воздухозаборник «холодного воздуха» на более новом двигателе с впрыском топлива, впускной патрубок может втягивать нагретый воздух из моторного отсека.Чтобы снизить риск детонации, вам нужен более прохладный и плотный воздух снаружи моторного отсека или перед радиатором, входящим в систему впуска.

Бедные топливные смеси могут вызывать детонацию . Богатые топливные смеси устойчивы к детонации, а бедные — нет. Утечки воздуха в вакуумных линиях, прокладках впускного коллектора, карбюраторах или прокладках корпуса дроссельной заслонки или прокладках впускного коллектора могут привести к попаданию дополнительного воздуха в двигатель. Бедные топливные смеси также могут быть вызваны загрязнением топливных форсунок, засорением форсунок карбюратора отложениями или грязью, засорением топливного фильтра или слабым топливным насосом.

Если топливная смесь становится слишком бедной, также могут возникать «пропуски зажигания на обедненной смеси», поскольку нагрузка на двигатель увеличивается. Это может вызвать колебания, спотыкание и грубый холостой ход.

На соотношение воздух / топливо также могут влиять изменения высоты. По мере того, как вы поднимаетесь вверх, воздух становится менее плотным. Карбюратор, который откалиброван для вождения на большой высоте, будет работать слишком бедно при движении на более низкой высоте. Изменение высоты, как правило, не является проблемой для карбюраторов с обратной связью последних моделей и электронного впрыска топлива, поскольку датчики кислорода и атмосферного давления компенсируют изменения плотности воздуха и соотношений топлива.

Поршень разрушен из-за преждевременного зажигания из-за того, что топливно-воздушная смесь стала слишком бедной при высокой нагрузке.

Неправильные свечи зажигания могут вызвать детонацию . Свечи зажигания с неправильным диапазоном нагрева (слишком горячие) могут вызвать детонацию, а также преждевременное зажигание. Свечи зажигания с медным сердечником имеют более широкий диапазон нагрева, чем обычные свечи зажигания, что снижает опасность детонации.

Потеря EGR может вызвать детонацию . Рециркуляция выхлопных газов (EGR) оказывает охлаждающее воздействие на температуру сгорания, поскольку она разбавляет поступающую смесь инертным выхлопным газом.Это снижает температуру горения и уменьшает образование оксидов азота (NOX). Это также снижает риск детонации. Таким образом, если клапан рециркуляции ОГ не работает или кто-то отсоединил его или засорил вакуумный шланг рециркуляции ОГ, температура сгорания будет намного выше, что может привести к детонации, когда двигатель находится под нагрузкой.

Чрезмерный турбонаддув может вызвать детонацию. Контроль количества наддува в двигателе с турбонаддувом абсолютно необходим для предотвращения детонации.Турбо-вестгейт сбрасывает давление наддува в ответ на повышение давления во впускном коллекторе. На большинстве последних моделей двигателей электромагнитный клапан с компьютерным управлением помогает регулировать работу вестгейта. Неисправность датчика давления в коллекторе, соленоида управления перепускной заслонкой, самой перепускной заслонки или утечка в вакуумных соединениях между этими компонентами может привести к тому, что турбонагнетатель обеспечит слишком большой наддув, что приведет к досрочной остановке двигателя, если состояние не будет исправлено. .

Улучшенное промежуточное охлаждение также может помочь.Работа интеркулера заключается в понижении температуры поступающего воздуха после того, как он выходит из турбокомпрессора. Добавление промежуточного охладителя к турбомотору, который не имеет промежуточного охлаждения, может устранить беспокойство о детонации, а также позволяет двигателю справляться с большим наддувом. А если заводской турбомотор был изменен, то для предотвращения детонации может потребоваться замена штатного промежуточного охладителя на более крупный и более эффективный промежуточный охладитель.

Неисправный датчик детонации может вызвать детонацию. Многие двигатели поздних моделей имеют «датчик детонации» на двигателе, который реагирует на частоту колебаний, характерных для детонации (обычно 6–8 кГц).Датчик детонации выдает сигнал напряжения, который сигнализирует компьютеру о необходимости на мгновение замедлить синхронизацию зажигания, пока детонация не прекратится. Датчик детонации обычно можно проверить, постучав гаечным ключом по коллектору или головке блока цилиндров рядом с датчиком (никогда не ударяйте по самому датчику!) И наблюдая за изменением времени, пока двигатель работает на холостом ходу. Если отсчет времени не удается замедлить, возможно, неисправен датчик или проблема может заключаться в электронной схеме управления синхронизацией зажигания самого компьютера.

Иногда датчик детонации реагирует на звуки, отличные от звуков детонации.Шумный механический топливный насос, неисправный водяной насос или подшипник генератора переменного тока или ослабленный шатунный подшипник — все это может вызывать вибрации, которые могут обмануть датчик детонации и заставить его замедлить время.

Проблемы детонации в двигателях с турбонаддувом и прямым впрыском

Некоторые поздние модели двигателей с турбонаддувом и прямым впрыском топлива могут испытывать детонацию на низких оборотах после холодного пуска или после продолжительного холостого хода. Проблема, по-видимому, связана с смешиванием бензина с остаточным моторным маслом на стенках цилиндров в верхней части цилиндра.Многие моторные масла содержат большое количество натрия в составе моющих присадок. Когда натрий смешивается с топливом, он образует соединение, которое может легко взорваться, когда двигатель сильно тянет под нагрузкой или ускоряется. Решение — перейти на моторное масло, которое содержит меньше моющего средства или меньше натрия в моющих присадках.





Статьи по теме:
Искровой детонатор

Рециркуляция выхлопных газов (EGR)

Плохой бензин может вызвать проблемы с производительностью

Обновление по плохому газу

Оценки с октановым числом топлива и рекомендации

Перегрев: причины и способы устранения

Нажмите здесь, чтобы увидеть больше технических статей Carley Automotive

.

В чем разница между детонацией и предварительным зажиганием?

Хорошо, я продолжаю читать о разнице между детонацией и предварительным зажиганием, но мне кажется, что всякий раз, когда я спрашиваю кого-нибудь об этом, они соглашаются, но не знают разницы. Типичный ответ, который я получаю: «Это действительно сложно». Так так ли это на самом деле?

J.M.

Джефф Смит: Это действительно не так уж и сложно объяснить. Детонация обычно является результатом любого количества комбинаций факторов.Во-первых, давайте разберемся, что происходит, когда происходит детонация, а затем выясним, почему это происходит, а затем разницу между детонацией и предварительным зажиганием.

Они совершенно разные, но в результате все равно может выйти из строя двигатель.

Это заблуждение относительно детонации — явление не новое. Откровенно говоря, двигатель внутреннего сгорания — очень сложный механизм, и когда мы начинаем его модифицировать, очень важно знать, как он работает. Я помню, как купил свою первую машину ’66 389 GTO, и она время от времени взрывалась или гремела.Я спросил старшего друга, который, как мне казалось, знал о двигателях, и он сказал: «О, не беспокойтесь об этом». С тех пор я узнал, что он не хотел признаваться, что не знал, но вам стоит об этом беспокоиться.

Начнем с процесса горения. В школе вас, возможно, учили, что когда загорается свеча зажигания , происходит большой взрыв, и все давление от этого взрыва толкает поршень вниз. Хотя это правда, что давление в цилиндре создается за счет расширения смеси воздуха и топлива, но на самом деле «взрыв» больше похож на огонь, горящий на пастбище из сухой травы.При измерении в миллисекундах процесс сгорания начинается в одной области или углу камеры сгорания и распространяется от свечи зажигания наружу. По мере продолжения процесса сгорания цилиндр нагревается по мере увеличения давления. Ближе к концу сгорания есть участки, где несгоревшее топливо теперь перегревается вместе с оставшимся кислородом. Если октановое число топлива достаточно низкое, эти отходящие газы будут стремиться к самовоспламенению, создавая скачок давления в цилиндре.Этот резкий скачок давления приводит к дребезжанию поршня в отверстии, смещению колец и, как правило, к передаче сильных ударных нагрузок на шатун , через верхнюю половину подшипника штока и на коленчатый вал . Этот удар мало чем отличается от удара молотка по поршню.

Это крупный план пары свечей зажигания, которые указали на неисправность в цилиндре. Вы можете ясно видеть эти черные точки на центральных электродах, но, возможно, более тревожным является то, что заземляющие электроды оплавлены.Исправление для этого двигателя заключалось в меньшем выборе времени (что снижает тепло в камере), непроектированной передней свече зажигания и более высоком октановом числе топлива.

Этого изображения должно быть достаточно, чтобы впечатлить вас, что слышимая (есть также случаи неслышимой или следовой детонации, которую вы не слышите) детонация — это плохо. Как правило, кованые поршни могут выдерживать короткие периоды детонации, как и некоторые литые поршни. Но литые поршни намного более хрупкие, и в зависимости от жесткости детонации могут возникнуть повреждения.Это еще хуже для двигателей с наддувом, с турбонаддувом или закиси азота, потому что давление в цилиндрах настолько выше, что ударные нагрузки намного выше по сравнению с двигателем без наддува. К настоящему времени должно быть ясно, что детонация — это плохо, и ее следует избегать. Один из способов узнать, произошла ли детонация в вашем двигателе, даже если вы ее не слышите, — это посмотреть на свечи зажигания. После того, как двигатель загремел, вы часто будете видеть крошечные черные пятна на фарфоре свечи зажигания.Я не могу объяснить, почему это происходит, но это достаточно распространено, поэтому, когда вы видите эти черные характеристики, пора внести изменения в настройку, либо убрав пару градусов опережения зажигания, либо увеличив октановое число топлива, либо и то, и другое. Еще одно относительно простое решение — снизить температуру воздуха на входе. Если вы когда-нибудь задумывались, почему двигатель, который перегревается, начинает детонировать, это потому, что температура воздуха на входе в камеру сгорания намного выше, потому что система охлаждения превратилась в пар, а также потому, что перегретый двигатель нагревает воздух, поступающий в камеру сгорания. камера сгорания.Конечный результат — детонация.

Предварительное зажигание — это совсем другое стечение обстоятельств. Вы можете определить причину этого по ярлыку. Как мы все знаем, процесс горения не начинается, пока искра не проскочит через межэлектродный зазор. Но с предварительным зажиганием что-то раскаленное в камере сгорания действует как свеча зажигания и воспламеняет смесь. Это когда начинают происходить плохие вещи. Часто это предварительное зажигание происходит, когда поршень находится глубоко в канале ствола и, возможно, сразу после закрытия впускного клапана.Поднимающийся поршень сжимает смесь, но, поскольку сгорание уже началось, через несколько миллисекунд давление в цилиндре достигает астрономического уровня, а поршень еще не достиг верхней мертвой точки (ВМТ). Конечным результатом является катастрофическое повреждение поршня и / или стенки цилиндра. Двигатель сразу же ломается без предупреждения или указания на то, что что-то пошло не так.

Это поршень одного из моих двигателей, у которого возникло преждевременное зажигание. Обратите внимание, что упорная сторона поршня отсутствует.Это не вина поршня. Вот что происходит, когда чрезмерное давление от предварительного зажигания ударяет по поршню. Одна из причин, по которой это сломалось, заключалась в том, что это заэвтектический поршень, который более хрупкий, чем поковка.

Раньше преждевременное зажигание было относительно редким явлением, но я думаю, что в последние несколько лет оно стало гораздо более распространенным, поскольку уровни мощности двигателя неуклонно растут. По моему опыту, большая часть причин преждевременного зажигания довольно проста — неправильный выбор свечи зажигания.Я испытал это на собственном опыте с двигателем Chevy с наддувом, который я тестировал. Это был , красиво построенный мотор с заэвтектическими поршнями, хороший плоский кулачок и хорошие головки. Одна из последних вещей, которые я добавил, — это свечи зажигания с удлиненным наконечником. Это были свечи холодного нагрева, но удлиненный носик свечи толкал ее дальше в камеру сгорания, что также означало, что заземляющая лента была длиннее. К сожалению, когда двигатель начал работать действительно хорошо, это означало, что в камере было больше тепла.Это дополнительное тепло также пошло на заземляющую планку. Поскольку ремешок длиннее, теплу требуется больше времени для передачи тепла в кожух свечи зажигания. На динамометрическом стенде на полной мощности после нескольких предыдущих прогонов, довольно близко друг к другу, двигатель внезапно потерял мощность. Мы заглушили двигатель и обнаружили, что заземляющий электрод свечи зажигания оплавился на нескольких свечах. Этого было недостаточно, чтобы вызвать резкую потерю мощности, но цилиндр также сильно опустился при сжатии коленчатого вала.Вкратце, когда мы разобрали двигатель, мы обнаружили, что поршень буквально сломан вместе с трещиной в стенке цилиндра. Вот что делает все избыточное давление. Честно говоря, должно было быть намного хуже.

Исправление для этого двигателя заключалось в том, чтобы использовать не только свечу диапазона холодного нагрева, но также использовать самую короткую свечу зажигания с центральным электродом, чтобы заземляющий провод был как можно короче. Это сокращает путь нагрева и должно предотвратить любые проблемы. Около года назад компания Autolite представила новую свечу типа X, которая представляет собой свечу зажигания с поверхностным зазором и двумя канавками, прорезанными на конце корпуса, создавая острые точки для проскока искры, при этом полностью устраняя заземляющий провод.Я провел несколько испытаний этой свечи с наддувом и закись азота, и результаты оказались многообещающими. Очевидно, у вас больше нет возможности считывать температуру на заземляющем ремешке для настройки. Но, по крайней мере, двигатель не загорится заранее.

Надеюсь, это объяснение проясняет очень реальные различия между детонацией и предварительным зажиганием.

Подводя итог всему этому, я проводил много времени с покойным Джоном Лингенфельтером в его магазине в Декейтере, штат Индиана. Ребята в магазине слушали рок-радиостанцию ​​из Саут-Бенда под названием The Bear.Раньше слоган станции был «103.9 — Детонационный камень!» Я всегда думал, что это ирония — стоять рядом с одним из двигателей Джона на динамометрическом стенде и слушать Detonation Rock, когда грохот двигателя был последним из , что мы хотели услышать.

Автор: Джефф Смит Джефф Смит страстно увлекался автомобилями с тех пор, как в 10 лет начал работать на заправочной станции своего деда. После окончания Университета штата Айова со степенью журналистики в 1978 году он объединил свои две страсти: автомобили и писательство.Смит начал писать для журнала Car Craft в 1979 году и стал редактором в 1984 году. В 1987 году он взял на себя роль редактора журнала Hot Rod, прежде чем вернуться к своей первой любви к написанию технических рассказов. С 2003 года Джефф занимал различные должности в Car Craft (включая редактора), написал книги о характеристиках автомобилей Small Block Chevy и даже собрал впечатляющую коллекцию Chevelles 1965 и 1966 годов. Теперь он регулярно пишет в OnAllCylinders. .

Устранение детонации: 9 способов предотвратить детонацию двигателя

(изображение любезно предоставлено Carboncleaningusa.com)

Detonation — отличная вещь, если вы смотрите шоу фейерверков или, возможно, смотрите MacGyver.

Внутри вашего двигателя? Не так много.

На самом деле, вероятно, будет лучше, если вы любой ценой избежите детонации в том, что касается вашего двигателя. Детонация возникает, когда из-за чрезмерного тепла и давления в камере сгорания топливно-воздушная смесь воспламеняется сама по себе.Вместо типичного единственного ядра пламени внутри камеры это создает множественное пламя, которое сталкивается со взрывной силой. Это вызывает резкое, внезапное повышение давления в цилиндре, в результате чего внутренние детали двигателя — поршни, кольца, подшипники, прокладки и т. Д. — подвергаются серьезной перегрузке и создают звук свистящего или стучащего звука. Худший сценарий: вы столкнулись с дорогостоящим, если не катастрофическим, повреждением двигателя.

Излишне говорить, что это не идеальная ситуация. Вот почему вместе с Summit Racing и Fel-Pro, мы составили список из девяти вещей, которые вы можете сделать, чтобы избежать проблемы с детонацией.

№1. Поднимите октановое число

Чем выше октановое число, тем лучше способность топлива противостоять детонации.

Большинство двигателей прекрасно работают на стандартном октановом числе 87; однако для двигателей с высокой степенью сжатия (9,0: 1 и выше) или с принудительной индукцией (нагнетатели или турбины) может потребоваться октановое число 89 или выше. Кроме того, приложения, в которых двигатель испытывает повышенную нагрузку или напряжение, например буксировка или тяжелая транспортировка, могут потребовать дополнительных уровней октанового числа. По сути, все, что вызывает более высокую температуру и давление сгорания или заставляет двигатель работать более горячим, чем обычно, может привести к детонации.

Возможно, пора поднять октановое число.

№2. Сохраняйте приемлемую степень сжатия

Статическое сжатие 9,0: 1 обычно является рекомендуемым пределом для уличных двигателей без наддува (хотя двигатели с датчиками детонации могут выдерживать более высокую степень сжатия). Для принудительной индукции может потребоваться статическое соотношение 8,0: 1 или меньше в зависимости от величины наддува. Степень сжатия более 10,5: 1 может вызвать детонацию даже при использовании бензина премиум-класса 93.

Уловка состоит в том, чтобы поддерживать степень сжатия в разумном диапазоне для перекачиваемого газа, если только ваш двигатель не построен для работы на гоночном топливе.Для этого вам может потребоваться использовать поршни с меньшей степенью сжатия, выбрать головки цилиндров с большими камерами сгорания или попробовать использовать прокладку под прокладку головки из меди с базовой прокладкой, чтобы уменьшить сжатие. Кроме того, если вы расточили цилиндры двигателя или фрезеровали головки цилиндров, это повысит степень сжатия, и вам, возможно, придется что-то делать.

№ 3. Проверьте свое время

Чрезмерная установка угла опережения зажигания может привести к слишком быстрому повышению давления в цилиндрах и, в конечном итоге, к детонации.Сбросьте время до заводских спецификаций. Если это не помогает, замедлите отсчет времени на пару градусов или попробуйте повторно откалибровать кривую опережения распределителя, чтобы контролировать детонацию.

№ 4. Управляйте своим ускорением

Регулировка количества наддува в двигателе с принудительным впуском имеет решающее значение.

Слишком большой наддув может привести к детонации, поэтому вам нужно либо А) уменьшить наддув, либо Б) оснастить двигатель, чтобы он выдерживал большее ускорение. Например, в системе с турбонаддувом вам необходимо убедиться, что ваш перепускной клапан работает правильно, чтобы стравить избыточное давление наддува.Утечки в вакуумных соединениях, неисправный датчик давления во впускном коллекторе или неэффективное управление соленоидом перепускной заслонки могут привести к тому, что турбонагнетатель будет выдавать слишком много наддува. Эти вещи следует исправить. И вы также можете добавить более производительный интеркулер , пока вы его используете.

Для применений с наддувом ознакомьтесь с нашими статьями по основам работы с воздуходувкой (Часть 2), и Основы работы с воздуходувкой (Часть 3) , чтобы узнать о надлежащих уровнях наддува и их отношении к сжатию.

№ 5. Наблюдать за смесью

Обедненные топливно-воздушные смеси склонны к детонации.

Проверьте свою топливно-воздушную смесь и отрегулируйте соответственно. Состояние обедненной смеси может быть признаком более серьезной проблемы, такой как утечки воздуха в вакуумных линиях или некачественные прокладки. Это также может быть вызвано грязными топливными форсунками , засоренными карбюраторными форсунками или засорением топливного фильтра. Если ваш двигатель испытывает колебания или грубую работу на холостом ходу, возможно, вы имеете дело с обедненным топливом, и вам нужно будет внести соответствующие регулировки или исправления до того, как произойдет детонация.

Нагар вокруг клапана. (Изображение любезно предоставлено carsandparts.com)

№6. Выдуть углерод

Углеродные отложения — частая причина детонации в двигателях с большим пробегом.

По сути, нагар может накапливаться в камере сгорания и на верхней части поршней до тех пор, пока не изменится общая компрессия двигателя. Кроме того, отложения могут создавать изолирующий эффект, который замедляет передачу тепла от камеры сгорания к головке цилиндров.Если отложения накапливаются достаточно (и сжатие увеличивается), может произойти детонация.

Как и указанная выше бедная топливная смесь, нагар может быть признаком другой проблемы: изношенных направляющих клапанов, износа цилиндров, поломки поршневых колец , или нечасто заменяемого масла. Выясните первопричину отложений, устраните все проблемы, а затем удалите отложения с помощью химического очистителя, металлической щетки или скребка (требуется удаление головок).

№ 7. Проверьте свой датчик детонации

Многие двигатели поздних моделей имеют датчик детонации , который может выйти из строя.

Датчик детонации реагирует на вибрацию в определенном частотном диапазоне. Когда частоты, которые обычно возникают при детонации, обнаруживаются, датчик детонации сообщает компьютеру транспортного средства о необходимости на мгновение замедлить зажигание, пока детонация не прекратится. В случае неисправности этот датчик перестанет работать.

Если на вашем автомобиле горит индикатор «Проверьте двигатель», возможно, у вас неисправный датчик детонации (среди прочего). Вы можете проверить бортовую компьютерную систему, прочитав код неисправности двигателя с помощью подходящих инструментов . Или вы можете проверить датчик детонации, постучав гаечным ключом по коллектору рядом с датчиком и наблюдая за изменением времени. Если отсчет времени не замедляется, датчик может быть неисправен. Вам нужно будет найти подходящую диагностическую таблицу в руководстве по обслуживанию вашего автомобиля, чтобы определить причину.

№ 8. Прочтите свои свечи зажигания

(Изображение любезно предоставлено Dynamicefi.com)

Обязательно прочтите наш предыдущий пост о , как читать свечи зажигания.

Вы можете многое сказать о характеристиках вашего двигателя, прочитав свои свечи.Например, если свечи зажигания выглядят желтоватыми, покрытыми пузырями или сломаны, они могут быть слишком горячими для применения. Попробуйте использовать свечи зажигания с более холодным диапазоном нагрева, чтобы избежать потенциальной детонации. Дополнительные советы см. В нашей публикации о диапазоне нагрева свечей зажигания .

№ 9. Подумайте о своей системе охлаждения

Если ваш двигатель перегревается, в нем больше шансов получить искровую детонацию. Вот почему вы должны убедиться, что ваша система охлаждения находится в хорошем состоянии.Проверьте уровень охлаждающей жидкости и при необходимости долейте. Убедитесь, что размер вашего вентилятора соответствует случаю. И обратите внимание на неисправный водяной насос, отсутствующий кожух вентилятора, слишком горячий термостат , проскальзывающую муфту вентилятора — практически все, что может помешать вашей системе охлаждения работать эффективно.

.

Зажигание

Аномалии свечей зажигания
  • Пилотный справочник по авиационным знаниям,
    Нормальное горение по сравнению с взрывоопасным горением
    • Детонация — это неконтролируемое взрывное воспламенение топливно-воздушной смеси в камере сгорания цилиндра.
    • Вызвано горячей температурой двигателя; или с использованием топлива с качеством ниже рекомендованного
    • Это вызывает чрезмерные температуры и давления, которые, если их не исправить, могут быстро привести к выходу из строя поршня, цилиндра или клапанов.
    • В менее тяжелых случаях детонация вызывает перегрев двигателя, неровности или потерю мощности
    • Характеризуется высокими температурами головки блока цилиндров и чаще всего возникает при работе на высоких настройках мощности
      • Использование более низкого сорта топлива, чем указано производителем самолета
      • Работа двигателя с очень высоким давлением в коллекторе в сочетании с низкими оборотами
      • Работа двигателя на высоких настройках мощности при слишком бедной смеси
      • Выполнение длительных наземных работ или крутых подъемов с пониженным охлаждением цилиндров
      • Убедитесь, что используется топливо надлежащего сорта
      • Держите створки капота (при наличии) в полностью открытом положении, когда находитесь на земле, чтобы обеспечить максимальный поток воздуха через капот
      • Используйте обогащенную топливную смесь, а также меньший угол набора высоты, чтобы улучшить охлаждение цилиндров во время взлета и начального набора высоты
      • Избегайте продолжительных крутых подъемов на большой мощности
      • Выработайте привычку контролировать приборы двигателя для проверки правильности работы в соответствии с процедурами, установленными производителем
    • Предварительное зажигание происходит, когда топливно-воздушная смесь воспламеняется до нормального воспламенения двигателя.
    • Преждевременное возгорание обычно вызывается остаточным горячим пятном в камере сгорания, часто возникающим из-за небольшого нагара на свече зажигания, треснувшего изолятора свечи зажигания или другого повреждения в цилиндре, которое вызывает нагрев детали достаточно для воспламенения заряд топлива / воздуха
    • Предварительное зажигание приводит к потере мощности двигателя и повышению рабочей температуры
    • Как и в случае детонации, предварительное зажигание может также вызвать серьезное повреждение двигателя, поскольку расширяющиеся газы оказывают чрезмерное давление на поршень, пока он все еще находится на его такте сжатия
  • Детонация и преждевременное воспламенение часто происходят одновременно, и одно может вызвать другое
  • Поскольку любое из этих условий вызывает высокую температуру двигателя, сопровождающуюся снижением производительности двигателя, часто бывает трудно отличить два
  • Использование рекомендованного сорта топлива и работа двигателя в надлежащих диапазонах температуры, давления и оборотов снижает вероятность детонации или преждевременного воспламенения.
  • Неисправности свечей зажигания
  • Справочник пилота по авиационным знаниям,
    Normal vs.Взрывное горение
    • В рамках контрольного списка выключения вы захотите проверить свои основные отведения (p-отведения) к магнето:
      • Это достигается быстрым перемещением ключа зажигания из ОБА в положение ВЫКЛ. (Или L, затем R в соответствии с процедурами), чтобы двигатель начал отключение.
      • Если двигатель не режет, значит, магнето не заземляется и поэтому «горячий»
      • Такое состояние может привести к запуску самолета только при движении винта, независимо от ключа в замке зажигания.
    • FADEC — это система, состоящая из цифрового компьютера и вспомогательных компонентов, которые управляют двигателем и воздушным винтом самолета
    • Впервые использованные в самолетах с турбинным двигателем и получившие название полностью авторитетного цифрового электронного управления, эти сложные системы управления все чаще используются в самолетах с поршневым двигателем.
    • В поршневом двигателе с искровым зажиганием FADEC использует датчики скорости, температуры и давления для контроля состояния каждого цилиндра.
    • Цифровой компьютер вычисляет идеальный импульс для каждой форсунки и регулирует момент зажигания по мере необходимости для достижения оптимальной производительности
    • В двигателе с воспламенением от сжатия FADEC работает аналогичным образом и выполняет все те же функции, за исключением тех, которые связаны с процессом искрового зажигания.
    • Системы
    • FADEC устраняют необходимость в магнето, обогревателе карбюратора, регуляторах смеси и заправке двигателя.
      • Одиночный рычаг дроссельной заслонки характерен для самолета, оснащенного системой FADEC
    • Пилот просто устанавливает рычаг дроссельной заслонки в нужное положение, такое как пуск, холостой ход, крейсерская мощность или максимальная мощность, а система FADEC автоматически настраивает двигатель и винт для выбранного режима
    • Во время запуска самолета FADEC заполняет цилиндры, регулирует смесь и устанавливает дроссельную заслонку в зависимости от температуры двигателя и давления окружающей среды
    • Во время крейсерского полета FADEC постоянно контролирует работу двигателя и регулирует расход топлива и угол зажигания индивидуально для каждого цилиндра.
      • Такой точный контроль процесса сгорания часто приводит к снижению расхода топлива и увеличению мощности
    • Должен быть доступен резервный источник электроэнергии, поскольку отказ системы FADEC может привести к полной потере тяги двигателя
    • Чтобы предотвратить потерю тяги, для резервирования включены два отдельных идентичных цифровых канала, каждый из которых может обеспечивать все функции двигателя и воздушного винта без ограничений.
    • Не нашли то, что искали? Продолжить поиск:

    .

    Как диагностировать ключ зажигания, который не поворачивает

    Может быть неприятно, когда вы вставляете ключ в замок зажигания автомобиля, а он отказывается поворачиваться. В вашем уме мчатся все возможности того, что может пойти не так, но, к счастью, большинство проблем с ключами зажигания не только обычны, но и быстро решаются. Есть три основных фактора, которые следует помнить при поиске причины, по которой ваш ключ не поворачивается, и с некоторыми устранениями неполадок эти советы могут помочь вам безопасно начать движение и уехать всего за несколько коротких шагов.

    Три основные причины, по которым ключ зажигания не поворачивается, включают: проблемы с соответствующими компонентами, проблемы с самим ключом и проблемы с цилиндром замка зажигания.

    • Совет : Всегда проверяйте, включен ли стояночный тормоз, чтобы автомобиль оставался в безопасности при выполнении этих шагов.

    Различные компоненты, относящиеся к системе зажигания, являются наиболее частыми виновниками того, что ключ вашего автомобиля не может повернуть зажигание. К счастью, их можно быстрее всего выявить и исправить.Следует знать о трех компонентах:

    Компонент 1: Рулевое колесо . Во многих автомобилях вынимание ключа блокирует поворот рулевого колеса. Иногда из-за этой блокировки рулевое колесо может застрять, что, в свою очередь, означает, что ключ автомобиля также застревает и не может двигаться, чтобы его отпустить. «Покачивание» рулевого колеса из стороны в сторону, одновременно пытаясь повернуть ключ, может ослабить это давление блокировки и позволить ключу повернуться.

    Деталь 2: Переключатель передач .Некоторые автомобили не позволяют ключу поворачиваться, если автомобиль не стоит на парковке или на нейтрали. Если автомобиль стоит на стоянке, слегка встряхните переключатель, чтобы убедиться, что он находится в правильном положении, и попробуйте еще раз повернуть ключ. Это касается только автомобилей с автоматической коробкой передач.

    Компонент 3: Аккумулятор . Если аккумулятор автомобиля разрядился, вы часто будете замечать, что ключ не поворачивается. В этом нет ничего необычного для автомобилей более высокого класса, которые часто используют более сложные электронные системы зажигания.Чтобы убедиться в этом, проверьте срок службы батареи.

    Причина 2 из 3: Проблемы с самим ключом

    Часто проблема не в компонентах автомобиля, а в самом ключе автомобиля. Следующие три фактора могут объяснить, почему ваш ключ не поворачивает замок зажигания:

    Фактор 1: гнутый ключ . Погнутые ключи иногда могут попасть в цилиндр зажигания, но, оказавшись внутри, они не выровняются правильно, чтобы позволить автомобилю завестись.Если ваш ключ выглядит погнутым, вы можете использовать неметаллический молоток, чтобы аккуратно разгладить его. Ваша цель — использовать что-то, что не повредит ключ, поэтому в идеале это будет резина или дерево. Вы также можете положить ключ на кусок дерева, чтобы смягчить удар. Затем очень осторожно нажимайте на ключ, пока он не встанет прямо, и попробуйте снова завести автомобиль.

    Фактор 2: Изношенный ключ . Изношенные ключи на самом деле очень распространены, особенно на старых автомобилях.Если ключ вашего автомобиля изношен, это не позволит штифтам внутри цилиндра упасть правильно и завести автомобиль. Если у вас есть запасной ключ, попробуйте сначала использовать его. Если вы этого не сделаете, вы можете получить запасной ключ, записав идентификационный номер автомобиля (VIN), который находится на лобовом стекле со стороны водителя или внутри дверного косяка. Затем вы захотите связаться с вашим дилером, чтобы изготовить новый ключ.

    • У некоторых новых автомобилей коды ключей прикреплены к набору ключей. Если ваш ключ изношен и вам нужен новый, вы можете передать этот код своему дилеру вместо VIN.

    Фактор 3: неправильный ключ . Иногда это простая ошибка, и в цилиндр вставляется не тот ключ. Чаще всего это случается, когда у кого-то на связке ключей больше одного ключа от машины. Многие клавиши выглядят одинаково, особенно если они одной марки. Поэтому дважды проверьте, используется ли правильный ключ, чтобы попытаться завести автомобиль.

    • Если вы видите, что ваш ключ грязный, его очистка также может помочь. Очистить сам ключ тоже очень легко.Используйте ватный тампон и медицинский спирт, чтобы удалить любые посторонние предметы, которые могут прилипнуть к ключу. Затем вы можете снова попытаться завести машину.

    • Некоторые ресурсы рекомендуют постучать по ключу молотком или другим предметом, когда он находится в замке зажигания, но это не рекомендуется из-за высокого риска не только поломки цилиндра, но и ключа. Это может привести к застреванию части ключа внутри цилиндра и еще большему повреждению.

    Причина 3 из 3: Проблемы с цилиндром замка зажигания

    Также известный как цилиндр ключа, цилиндр замка зажигания — еще одна область, которая может вызывать проблемы с переворачиванием ключа.Ниже приведены две наиболее распространенные проблемы, связанные с цилиндром замка зажигания и не поворачивающимся ключом.

    Проблема 1: Препятствие . Препятствие внутри цилиндра ключа не позволит ключу правильно повернуть зажигание. Загляните внутрь на цилиндр ключа с фонариком. Вам нужно будет искать любые очевидные препятствия. Иногда, когда ключевой цилиндр полностью выходит из строя, вы можете увидеть внутри металлический мусор.

    • Если вы пытаетесь очистить цилиндр замка зажигания, всегда начинайте с защитных очков, чтобы защитить глаза от любого летящего мусора.Для очистки используйте электрический очиститель или сжатый воздух и соблюдайте меры предосторожности и инструкции, указанные на баллончике. Убедитесь, что ваше рабочее место хорошо вентилируется. При необходимости можно попробовать произвести повторное распыление. Если мусор был успешно удален, ключ должен легче войти.

    Проблема 2: Заедание пружин . Штифты и пружины внутри ключевого цилиндра соответствуют уникальной форме вашего ключа, так что только ваш ключ будет работать, чтобы включить вашу машину. Могут возникнуть проблемы с поворотом ключа из-за проблем со штифтами или пружинами.В этом случае используйте небольшой молоток, чтобы осторожно постучать по ключу зажигания. Это поможет ослабить застрявшие штифты или пружины. Вы не хотите сильно бить — цель состоит в том, чтобы использовать вибрацию крана, а не силу, чтобы ослабить застрявшие штифты или пружины. Когда они освободятся, вы можете попытаться вставить ключ и повернуть его.

    Перечисленные выше методы — отличный способ повернуть ключ, если он отказывается сдвинуться с места. Однако, если вы все еще боретесь с ключевыми проблемами поворота после того, как попробуете все эти советы, вам следует обратиться к механику для дальнейшей диагностики.YourMechanic предоставляет сертифицированных мобильных механиков, которые приходят к вам домой или в офис и могут легко диагностировать, почему ваш ключ не поворачивается, и произвести необходимый ремонт.

    .

    Почему происходит детонация двигателя при выключении зажигания: основные причины « NewNiva.ru

    На что влияют неисправности датчика детонации

    Можно ли ездить с неисправным датчиком детонации? Этот вопрос интересует автолюбителей, впервые столкнувшихся с данной проблемой. В общих чертах ответ на этот вопрос можно сформулировать так — в краткосрочной перспективе автомобилем пользоваться можно, однако при ближайшей же возможности необходимо провести соответствующие диагностику и устранить проблему.

    Ведь по принципу работы ЭБУ когда возникает неисправность датчика детонации топлива, то автоматически устанавливается позднее зажигание чтобы исключить повреждение деталей поршневой группы при возникновении реального детонирования при сгорании топливной смеси. Как результат — поднимается расход топлива и значительно падает динамика которая особенно станет заметной при повышении оборотов.

    Бедная смесь

    Обедненная рабочая смесь способствует детонации, потому что меньшее количество испаренного топлива, поглощает меньшее коли­чество теплоты. Таким образом, бедная смесь увеличивает температуту — главную причины детонации. Двигатель с турбонаддувом требует слегка более богатых смесей, чем обычный атмосферный двига­тель, при этом дополнительное топливо действует подобно жидкому промежуточному охладителю. Назовем это штатным промежуточным охладителем.

    Где находится датчик детонации нива шевроле и его неисправности

    Одним из наиболее популярных отечественных легковых автомобилей является Нива Шевроле. Эта полноприводная машина имеет бензиновый двигатель и относится к внедорожникам. Исходя из этого, Нива обладает склонностью к частым поездкам по ухабам и бездорожью. Условия эксплуатации данного транспортного средства располагают к обеспечению максимальной защиты двигателя.

    Наиболее частые причины детонации двигателя и способы решения

    Настройка зажигания

    Неправильная установка угла опережения зажигания — это скорее не неисправность системы, а неточность ее регулировки. Проверка статического и максимального угла опережения зажигания практически всегда покажет имеющееся несоответствие в системе зажигания. Система управления углом опережения зажигания по датчику детонации может быть причиной многих неисправностей, одна из ко­торых — ошибка в распознавании детонации и действиях по ее устране­нию.

       Если возможна неисправность датчика детонации, обратитесь к инструкции по эксплуатации системы или, в случае штатной системы, к инструкции к транспортному средству.

    Неисправность системы охлаждения

    Также топливо детонирует, если в силовом узле неисправна охлаждающая система. При такой неполадке наблюдается детонация двигателя при разгоне. Под нагрузкой мотор перегревается, внутреннее пространство камеры сгорания раскаляется до температуры, когда пары бензина самовоспламеняются.

    Неисправные свечи

    Иногда причина детонации двигателя ВАЗ или другой марки автомобиля – неисправные или неподходящие по параметрам свечи зажигания. В этом случае искра может генерироваться не так, как рассчитывал производитель мотора. Несвоевременное искрение свечи – одна из распространенных причин проблем воспламенения топливовоздушной смеси.

    Неправильно выставленное зажигание

    Если детонация двигателя появилась вслед за попыткой отрегулировать угол зажигания, то причина в неправильной настройке. Даже мастера в автомастерской могут ошибаться, тем более ошибка возможна при неквалифицированном вмешательстве.

    Обращайтесь только в проверенные технические центры и очень осторожно относитесь к советам изменить опережение зажигания. Лучше вообще не трогать настройки завода производителя, если нет полной уверенности, что они сбились.

    То же самое можно сказать о манипуляциях с обеднением топливовоздушной смеси. Часто эта операция приводит к возникновению детонации двигателя ВАЗ, УАЗ или автомобилей других марок. Семь раз подумайте, прежде чем изменять заводские настройки.

    Неправильно настроенное зажигание

    Стремясь повысить крутящий момент, некоторые умельцы изменяют заводские настройки системы зажигания. Если выставить слишком большой угол опережения, свеча будет давать искру раньше, чем поршень приблизится к ВМТ. Воспламенение произойдет раньше времени, когда горючее не полностью перемешалось с воздухом.

    Обедненная топливовоздушная смесь

    В погоне за экономичностью автомобилисты могут специально обеднять топливовоздушную смесь. Это еще одна причина, почему возникает детонация двигателя. Из-за недостаточной концентрации паров горючего искра не может воспламенить смесь. При следующем цикле впрыска, наоборот, паров топлива становится больше нормы. Чрезмерно обогащенная смесь воспламеняется от сжатия раньше времени.

    Октановое число

    Самый вероятная причина возникновения детонации — октановое число топлива. Зто — критерий стойкости горючего к самовозго­ранию, или детонации. Чем больше октановое число, тем выше эта стой­кость. Качество топлива относительно постоянно, но когда оно под сомнением, желательно сменить марку топлива, или перейти на более высокое октановое число.

    Принцип работы двигателя

    Детонация очень часто проявляется при подъеме в горку. При этом автомобиль двигается медленно, а скорость включена повышенная. Но не стоит переживать, так как это явление называется калильным зажиганием, и то он не свидетельствует о том, что двигатель работает неправильно.

    Любой двигатель внутреннего сгорания работает по таким принципам:

    1. Смесь из бензина и воздуха подается в камеру сгорания цилиндров, где она поджигается при помощи искры на свече зажигания. Причём поршень должен находиться буквально в нескольких миллиметрах от верхней мертвой точки. Как только поршень достигнет верхнего положения, в цилиндре образуется наибольшее давление. В этот момент вся топливовоздушная смесь полностью сгорит.
    2. При нормальной работе датчика детонации на «Шевроле Нива» признаки этого явления не будут проявляться. Этот прибор поможет избавиться от проявление детонации. А появляется она, когда смесь начинает воспламеняться намного раньше, нежели положено. Обычно это происходит на половине пути до верхней мертвой точки. И при сгорании топливовоздушной смеси на верхнюю часть поршня воздействует сила, которая толкает его в обратную сторону. При этом снижается, причём существенно, мощность двигателя, и изнашиваются его детали.
    1. Смесь из бензина и воздуха подается в камеру сгорания цилиндров, где она поджигается при помощи искры на свече зажигания. Причём поршень должен находиться буквально в нескольких миллиметрах от верхней мертвой точки. Как только поршень достигнет верхнего положения, в цилиндре образуется наибольшее давление. В этот момент вся топливовоздушная смесь полностью сгорит.
    2. При нормальной работе датчика детонации на «Шевроле Нива» признаки этого явления не будут проявляться. Этот прибор поможет избавиться от проявление детонации. А появляется она, когда смесь начинает воспламеняться намного раньше, нежели положено. Обычно это происходит на половине пути до верхней мертвой точки. И при сгорании топливовоздушной смеси на верхнюю часть поршня воздействует сила, которая толкает его в обратную сторону. При этом снижается, причём существенно, мощность двигателя, и изнашиваются его детали.

    Промежуточный охладитель (например интеркуллер)

    Промежуточный охладитель серьезно влияет на порог детонации двигателя с турбонаддувом. Что-нибудь, что снижает эффективность промежуточного охладителя, понижает порог детонации. Кроме удаления различного мусора, вроде клочков газет, застрявшего перед про­межуточным охладителем, единственное необходимое периодическое обслуживание состоит в удалении масляного налета изнутри, который накапливается при нормальной эксплуатации. Масляный налет заметно уменьшает эффективность промежуточного охладителя.

    Противодавление отработанных газов

    Слишком маленькая турбина будет блокировать выхлопные газы в выпускном коллекторе, или является, в некотором смысле, сужением в выхлопной системе и приведет к увеличению обратного давления в системе. Противодавление удерживает горячие газы в камере сгорания. Неисправность любого вида, которая увеличивает противодавление, серьезно ухудшает детонационную стойкость двигателя.

    Температура окружающей среды

    Бывают дни, когда все работает не так как должно, и температура окружающей среды, конечно, вносит в это свой вклад. Системы турбо­наддува, работающие с высокими уровнями наддува, обычно работают на грани порога детонации и могут легко преодолеть эту страшную гра­ницу, если температура окружающей среды изменится в худшую сто­рону.

    Заключение

    Именно поэтому при появлении первых симптомов детонации необходимо проводить диагностику двигателя и принимать меры по устранению неисправности. Чистого всем бензина и ровных дорог!

    Читать новости о новой Ниве

    Признак

    Неисправность

    Причина

    Решение

    Детонация появилась после заправки.

    Топливовоздушная смесь самовоспламеняется.

    Низкокачественное топливо с неподходящим октановым числом.

    Слить топливо, промыть двигатель промывкой «Супротек».

    Детонация сразу после запуска мотора.

    Искра слишком рано поджигает топливовоздушную смесь.

    Неправильно настроенное зажигание.

    Отрегулировать угол зажигания.

    Детонация в любом режиме работы.

    Обедненная топливовоздушная смесь.

    Неправильно настроен впрыск.

    Отрегулировать впрыск топлива.

    Детонация после выключения зажигания.

    Топливовоздушная смесь самовоспламеняется без искры.

    Нагар на стенках цилиндров.

    Использовать промывку или присадку SGA от Suprotec.

    Детонация в любом режиме работы.

    Неподходящие или неисправные свечи.

    Несвоевременный поджиг топливовоздушной смеси.

    Заменить свечи.

    Детонация под нагрузкой.

    Перегрев силового агрегата.

    Неисправность системы охлаждения двигателя.

    Проверить исправность всех компонентов системы охлаждения.

    Детонация появляется во время долгой поездки.

    Перегрев силового агрегата.

    Неисправность системы охлаждения двигателя.

    Проверить исправность всех компонентов системы охлаждения.

    При детонации выхлоп черного или зеленого цвета.

    Частицы алюминия в продуктах сгорания топлива.

    Разрушение компонентов двигателя.

    Полная замена ЦПГ и других поврежденных деталей.

    Детонация двигателя

    Автор admin На чтение 6 мин. Просмотров 101

    Такое явление, как детонация двигателя, знакомо практически каждому автовладельцу. Чаще всего она возникает при движении в гору на высокой передаче с небольшой скоростью. К звуку работы двигателя внутреннего сгорания (ДВС) примешивается жесткий металлический стук, который многие принимают за стук поршневых пальцев.

    Что такое детонация

    Детонация – это процесс взрывного воспламенения рабочей смеси в цилиндрах двигателя. В то время как нормальная скорость распространения фронта пламени составляет около 30 м/с, при детонации огонь распространяется в десятки раз быстрее – до 2000 м/с.


    В нормальных условиях смесь начинает воспламеняться, когда поршень немного не доходит до верхней мертвой точки, угол опережения зажигания составляет обычно 2-3 градуса. Завершается вспышка после того, как поршень минует ВМТ. В случае детонации смесь воспламеняется еще в середине такта сжатия. Поршень испытывает сильное противодействие, в итоге пропадает мощность двигателя и значительно повышается расход топлива.

    Данное явление никогда не идет на пользу мотору, однако детонацию можно разделить на допустимую и недопустимую. В первом случае ее даже не всегда удается заметить. Обычно она возникает на низких оборотах и продолжается недолго. Чаще всего подобное происходит в двигателях небольшого объема с относительно большой мощностью и крутящим моментом (например, 107 л.с. и 135 Нм при объеме 1,4 л). Недопустимая детонация, как правило, возникает в форсированных ДВС при повышенных нагрузках на высоких оборотах. Всего после нескольких секунд работы в таких условиях, мотор может получить критические повреждения.

    Существует еще одно явление, которое автовладельцы нередко путают с детонацией – дизелинг. Мотор после выключения зажигания продолжает работать рывками, то с повышением, то с понижением оборотов, звук работы двигателя при этом металлический, схожий со звуком детонации. Это явление иного рода и причины его появления иные: при глушении мотора, бензин в цилиндрах самовоспламеняется из-за высокой степени сжатия, как в дизельном ДВС, отсюда и название. Не следует путать дизелинг с калильным зажиганием – там при глушении рабочая смесь воспламеняется от нагретых электродов свечей и нагара.

    Чем опасна детонация

    Весь кривошипно-шатунный механизм и головка блока цилиндров испытывают разрушающие нагрузки, способные при длительном воздействии привести к поломке ДВС. Кроме того, температура в цилиндрах также поднимается до недопустимых значений (до +3700 градусов), что грозит прогаром прокладки ГБЦ, а также коррозией днища поршня и зеркала цилиндров.


    Прокладка головки блока – это первая деталь, которая придет в негодность из-за детонации. Она способна перенести лишь кратковременную работу в режиме запредельных термических и механических нагрузок. Худшее, чем грозит детонация – замена блока цилиндров, коленчатого вала, поршневой группы и головки блока.

    Причины возникновения детонации

    Причины, в силу которых возникает данное явление, можно разделить на три группы:

    • октановое число бензина;
    • конструктивные особенности ДВС;
    • условия эксплуатации автомобиля.

    Влияние октанового числа

    В отличие от дизельного двигателя, в котором воспламенение рабочей смеси происходит благодаря высокой степени сжатия, в бензиновом для этой цели применяется система зажигания. Смесь бензина и воздуха поджигается искрой, возникающей между электродами свечей.

    Степень сжатия у бензиновых моторов намного меньше, это связано с тем, что бензин не столь устойчив к детонации, как дизельное топливо. Основной характеристикой бензина является октановое число, отражающее его детонационную стойкость. Чем оно выше, тем сильнее можно сжать топливно-воздушную смесь.

    Если автомобиль, силовой агрегат которого рассчитан на применение топлива с октановым числом не ниже 95, заправить бензином марки АИ-92, то с высокой долей вероятности можно утверждать, что при высоких нагрузках рабочая смесь в цилиндрах будет детонировать.

    Однако проблема может появиться и в случае, если марка топлива соответствует рекомендациям производителя. Все дело в качестве бензина. Недобросовестные продавцы нередко самостоятельно повышают октановое число, путем добавления в горючее сжиженного пропана или метана. Эти газы очень быстро испаряются, после чего в баке остается низкооктановый бензин.


    Вследствие детонации низкооктанового топлива, в камере сгорания усиленно образуется нагар, который, в свою очередь, может вызвать такое явление, как калильное зажигание. В этом случае двигатель продолжает работать даже после выключения зажигания. Причины его возникновения в том, что воспламеняется топливно-воздушная смесь не от искры, а от раскаленных электродов свечи или нагара.

    Влияние конструктивных особенностей

    Причины возникновения детонации могут крыться в конструктивных особенностях двигателя.
    К их числу можно отнести:

    1. степень сжатия;
    2. форму камеры сгорания;
    3. форму днища поршня;
    4. наличие наддува;
    5. расположение свечей зажигания.

    Так, чем выше степень сжатия, тем ДВС более склонен к детонации. То же можно сказать и о системах наддува («надутым» моторам требуется высокооктановый бензин).

    Влияние условий эксплуатации

    Не последнюю роль играют и условия, в которых эксплуатируется машина. Детонация может возникать при движении на повышенной передаче с низкой скоростью. Так, если попытаться въехать в гору на четвертой передаче со скоростью 30 км/ч, из-под капота незамедлительно раздастся характерный металлический стук.

    Свое влияние оказывает правильность работы системы зажигания (рабочая смесь в цилиндрах детонирует при раннем зажигании), исправность системы охлаждения двигателя, наличие нагара на поршнях и в камерах сгорания. Подвергают себя опасности автовладельцы, стремящиеся любыми способами уменьшить аппетит машины. С этой целью электронный блок управления «перепрошивается» для приготовления более бедной смеси, чем нужно. В результате ухудшается динамика авто, а при повышенных нагрузках возникает детонация.

    Способы борьбы с детонацией

    Для того чтобы устранить данное явление, необходимо обратить внимание на причины его возникновения и помнить, что детонация происходит при включенном зажигании, ненормальные явления, возникающие при глушении мотора, имеют иное название и требует иных мер.

    Если ДВС стал работать с детонацией сразу после заправки – значит, в бак попало некачественное горючее. Если мотор бензиновый, можно добавить в топливный бак ацетон, он повысит октановое число. Либо топливо придется попросту слить и заправиться более качественным.

    Если автомобиль постоянно эксплуатируется в щадящих условиях, с минимальной нагрузкой или же мотор подолгу работает на холостом ходу, в камерах сгорания откладывается нагар, из-за чего повышается степень сжатия и увеличивается риск появления детонации. В данном случае необходима профилактика: двигателю необходимо периодически давать работать с большой нагрузкой. Хороший метод профилактики – периодические динамичные разгоны (что называется «тапка в пол») и езда на пониженной передаче с высокими оборотами. Разумеется, это допустимо только если позволяет обстановка на дороге.


    Детонация дизельного ДВС иногда сопровождается черным или зеленоватым выхлопом. Это означает, что разрушились поршни, и выхлопным газам добавляются частицы алюминия. В такой ситуации необходима замена поршневой группы.

    Из-за неисправных свечей зажигания может возникать детонация при запуске двигателя. В этом случае все, что нужно сделать – заменить их. У дизельного двигателя такая проблема возникает после западания иглы форсунки. Чтобы устранить неполадку, придется посетить СТО.

    Мне нравится1Не нравится
    Что еще стоит почитать

    Детонация двигателя

    Детонация — это самовозгорание внутри цилиндра ПОСЛЕ возгорания свечи зажигания. Он похож на предварительное зажигание, но отличается.

    При нормальном зажигании свеча зажигания зажигается непосредственно перед достижением поршнем ВМТ. Пламя проходит через камеру сгорания, воспламеняя смесь воздух / топливо. Это вызывает постоянное увеличение давления в цилиндре и вынуждает поршень опускаться при рабочем ходе.

    Когда происходит детонация, часть воздуха / топлива воспламеняется до того, как до него дойдет нормальное горение.Это вызывает кратковременный, но сильный скачок давления в цилиндре.

    Детонацию также называют «стуком двигателя», «стуком» или «звоном» из-за издаваемого звука.

    Как это обозначено?

    • Звук стука или звона
    • Падение температуры выхлопных газов (EGT)
    • Разбитые поршневые кольца и / или свечи зажигания
    • Повреждение поршня и / или клапанов

    Что его вызывает?

    Детонация может быть вызвана несколькими факторами.Несколько распространенных причин:

    Усовершенствованная синхронизация зажигания
    Если синхронизация зажигания слишком велика, свеча зажигания загорается слишком быстро. Это приводит к преждевременному прекращению пламени. Оставшееся топливо может взорваться.

    Обедненная смесь воздуха и топлива
    Богатая смесь воздуха и топлива работает холоднее, чем бедная смесь. Нежирная смесь может стать слишком горячей и взорваться.

    Слишком сильное сжатие
    Сжатие вызывает нагревание. Если топливно-воздушная смесь сжата слишком сильно, она может взорваться.

    Перегрев двигателя
    Низкий уровень охлаждающей жидкости или неисправный водяной насос могут вызвать перегрев двигателя. Слишком большое количество тепла может вызвать детонацию воздуха / топлива в камере.

    Низкооктановое топливо
    Октановое число является мерой «детонационной стойкости». Переход на более качественное топливо может помочь при детонации двигателя.

    Как это влияет на производительность?

    Двигатель разработан для работы определенным образом. Поскольку детонация нарушает эту конструкцию, она лишает двигатель мощности.

    Большинство двигателей выдерживают небольшую детонацию. Современные двигатели с впрыском топлива могут распознавать детонацию и регулировать соотношение воздух / топливо и время зажигания. Однако, если детонация не зафиксирована, это приведет к повреждению двигателя. Всего один крупный взрыв может нанести значительный ущерб.

    ID ответа 5007 | Опубликовано 30.05.2018 12:58 | Обновлено 12.11.2019 14:46

    Что такое детонация и 8 способов ее остановить!

    Детонация — это ругательство вокруг хот-родов.Никто не любит говорить об этом, потому что, когда это происходит, обычно означает некоторую упущение внимания во время сборки двигателя или автомобиля. К тому времени, когда вы услышите характерный предсмертный хрип двигателя в агонии взрыва, ущерб, скорее всего, уже нанесен. Ответ состоит в том, чтобы предотвратить детонацию до того, как это произойдет, но если будет слишком поздно, есть вещи, которые вы можете сделать, чтобы предотвратить повторное выполнение, но сначала немного предыстории.

    Каковы симптомы детонации?

    Детонация — иногда называемая детонацией или преждевременным зажиганием — это свистящий звук, который иногда можно услышать во время разгона и открытия дроссельной заслонки.В отличие от обычного шума выхлопных газов, детонация — это высокий скрипучий звук, который исходит из моторного отсека. В случае детонации может произойти серьезное внутреннее повреждение, в том числе оплавленные электроды свечи зажигания, треснувшие поршневые кольца, оплавленные или треснувшие поршни, забитые подшипники штока и взорванные прокладки головки. Если вы услышите детонацию, немедленно уберите ногу с дроссельной заслонки или заплатите за последствия.

    Просмотреть все 12 фото

    Повреждение от детонации происходит из-за того, что головка поршня, кольца и подшипники подвергаются сильному избыточному давлению в камере сгорания.Это избыточное давление возникает слишком рано, задолго до того, как поршень начнет движение вниз для рабочего хода. Это повышение давления во время такта сжатия также выделяет огромное количество тепла — на самом деле, слишком много для того, чтобы система охлаждения двигателя могла вовремя рассеяться. Затем каждое последующее срабатывание этого цилиндра должно бороться с остаточным теплом от события детонации в предыдущем цикле, таким образом блокируя рабочее состояние этого цилиндра от безудержной детонации.

    Если у вас когда-либо был случай детонации двигателя, вы уже знакомы с этим явлением.Если снять ногу с педали газа, а затем снова вставить ее, детонация, исходящая от двигателя, не улучшится; он остается там до тех пор, пока не исчезнет нежелательный источник возгорания (тепло) в пораженном цилиндре. Если посмотреть с другой стороны, если в определенном рабочем состоянии начало детонации происходит, например, при 15 градусах перед верхней мертвой точкой (BTC), она может не прекратиться, пока событие воспламенения не будет отложено до 5 градусов BTC. Такое поведение называется гистерезисом детонации, и ваш единственный реальный вывод здесь состоит в том, что мгновенный сброс газа (в надежде, что детонация исчезнет) — бесплодная попытка.

    Что вызывает детонацию?

    Тип повреждения двигателя, вызванный детонацией, происходит, когда источник тепла в среде сгорания воспламеняет топливно-воздушный заряд до инициирования системой зажигания двигателя. Важно понимать, что детонация является результатом нежелательного источника тепла (электрод свечи зажигания, края камеры сгорания, неровности литья), а не ошибки в программировании зажигания, хотя ваша программа зажигания может сыграть свою роль.

    Просмотреть все 12 фотографий

    В двигателе с оптимизированными характеристиками пиковая мощность достигается, когда давление в цилиндре достигает максимума при правильном угле поворота коленчатого вала.Когда шатун и коленчатый вал расположены под углом 90 градусов друг к другу, поршень имеет наибольшее механическое преимущество по отношению к коленчатому валу. Смысл всего этого в том, что вам нужно мысленно вернуться к этому событию и найти подходящее время для воспламенения топливного заряда, чтобы пиковое давление произошло после прохождения поршнем верхней мертвой точки (ВМТ) и до того, как ход штока и кривошипа достигнет под прямым углом. Несоблюдение этого требования может привести к повреждению поршневого кольца, как показано выше.

    Когда давление в цилиндре достигает пика перед ВМТ, случаются неприятности. Почему? Это может быть одна или несколько из следующих причин: слишком низкое октановое число топлива, недостаточная система охлаждения, плохо спроектированная камера сгорания, слишком горячая свеча зажигания, слишком большое статическое сжатие, слишком маленькое перекрытие между впускным и выпускным лепестками кулачка, слишком бедное соотношение воздух / топливо, слишком сильный предварительный нагрев всасываемого заряда или, в лучшем случае, неправильная кривая зажигания.

    8 способов уменьшить детонацию

    Если ваш двигатель испытывает детонацию, вы можете предпринять ряд действий, чтобы предотвратить ее.Здесь мы расположили их в порядке сложности, от самого простого до самого серьезного, но имейте в виду, что часто детонация и наносимый ею ущерб являются результатом плохо выбранной комбинации двигателей. Производители оригинальных комплектующих тратят тысячи часов на испытания двигателей в различных режимах работы, в то время как при самостоятельной работе можно упускать из виду важные аспекты, такие как качество движения смеси или тщательный учет времени срабатывания клапана. Эти вещи должны быть вплетены в конструкцию двигателя перед сборкой, а не закреплены бандажом постфактум.

    Посмотреть все 12 фото

    Уменьшите время опережения зажигания

    Если вам повезет, ваша детонация будет вызвана не самовоспламенением от горячей точки в камере сгорания, а кривой зажигания, которая обеспечивает слишком сильное базовое воспламенение продвигать. В этом случае простое уменьшение базовой синхронизации приведет к прекращению стука. Однако в большинстве случаев причиной этого состояния в первую очередь будет отключение подачи вакуума. В этом сценарии задействован энтузиаст-новичок, который отключает подачу вакуума, а затем увеличивает базовое время для компенсации.Вся причина увеличения вакуума на серийном двигателе состоит в том, чтобы обеспечить достаточное время выполнения заказа в условиях небольшого дросселя, когда атмосфера за дроссельной заслонкой тонкая; двигателю требуется дополнительное время для повышения давления в цилиндре перед рабочим ходом.

    Посмотреть все 12 фотографий

    Увеличьте октановое число топлива

    Октановое число топлива является точным выражением его склонности к самовоспламенению. Чем выше число, тем выше его способность противостоять выключению света. По мере увеличения степени сжатия или наддува должно возрасти октановое число топлива.Устранить детонацию в двигателе можно так же просто, как использовать топливо с более высоким октановым числом. В 1970-х и 1980-х годах, когда цены на топливо резко выросли, многие люди искали способы сэкономить деньги. Это часто выражалось в снижении октанового числа топлива. К счастью для нас, с тех пор под мостом прошло много воды, и производители разработали двигатели с улучшенными противодетонационными характеристиками. Такие вещи, как электронный впрыск топлива, замедление детонации и электроника, определяющая октановое число, сделали детонацию из-за низкого октанового числа топлива редким явлением.Получил старую машину с детонацией, попробуйте запустить тестовое топливо с более высоким октановым числом.

    Просмотреть все 12 фото

    Используйте более холодную свечу зажигания

    Наконечник электрода свечи зажигания является основным источником самовоспламенения. Тепло может быстро накапливаться, и если ему некуда деваться, он сделает свое дело с зарядом воздуха / топлива. Звучит немного иронично, поскольку это то, что должна делать свеча зажигания, только вы хотите иметь контроль над , когда это делает. По этой причине свечи зажигания рассчитаны на различные диапазоны нагрева, а их изоляторы тщательно разработаны для управления потоком тепла от электрода в головку блока цилиндров.Слишком горячая свеча будет удерживать слишком много тепла, вызывая детонацию. Замена свечи на более холодный нагревательный элемент — это часто все, что нужно для отключения нежелательной детонации. Однако имейте в виду, что слишком холодная свеча может вызвать обратную проблему — засорение, когда свеча не может полностью воспламенить воздушный / топливный заряд.

    Просмотреть все 12 фотографий

    Оптимизация соотношения воздух / топливо

    В современных двигателях с впрыском топлива достижение оптимального соотношения воздух / топливо редко является проблемой, так как кислородные датчики двигателя будут постоянно работать, сохраняя воздух / соотношение топлива в идеальном диапазоне в большинстве сценариев вождения.Однако более старым карбюраторным автомобилям может потребоваться помощь, особенно если детонация является регулярной проблемой. Здесь проблема заключается в обедненной смеси, когда впрыскивание или какая-либо другая калибровка приводит к тому, что в цилиндр не поступает достаточно топлива. В результате получается горячий двигатель, который быстро нагревается и может вызвать детонацию. Лучший способ диагностировать детонацию в этой ситуации — установить широкополосный датчик кислорода и контролировать его в периоды высокой нагрузки двигателя. Обедненная смесь при полном открытии дроссельной заслонки может вызвать детонацию даже при соотношении 13: 1, и это должно указывать на то, что в цилиндр поступает недостаточно топлива.Вы должны убедиться, что ваш двигатель работает на разогретой скорости — хорошее значение для двигателя без наддува составляет 12,5: 1 при полном открытии дроссельной заслонки.

    Просмотреть все 12 фотографий

    Увеличить охлаждающую способность

    Тепло является основной причиной детонации, и часто одной из основных причин является неэффективная система охлаждения. Если мощность вашего двигателя недавно была увеличена, но ваш радиатор все еще в наличии, возможно, пришло время для модернизации в этой области. За исключением замены радиатора, более эффективный вентилятор, более эффективный кожух вентилятора или смачивающий агент охлаждающей жидкости могут иметь достаточный эффект для уменьшения или устранения детонации.Объяснение диагностики системы охлаждения выходит за рамки этой истории, но мы оставим вам один большой совет: сначала подумайте о мелочах. В гонке за производительностью часто к детонации приводят такие мелочи, как уплотнение кожуха, работа термостата, включение муфты вентилятора или кавитация водяного насоса.

    Просмотреть все 12 фотографий

    Уменьшите степень сжатия

    Если вы зашли так далеко, но все еще есть детонация, у вас не будет другого выхода, кроме как начать внутренний ремонт или изменить комбинацию двигателей.Уменьшение степени сжатия — это самый простой способ остановить детонацию, потому что давление и тепло в цилиндре — это, по сути, разные выражения одного и того же. На протяжении многих лет мы приводили доводы в пользу увеличения степени сжатия для увеличения мощности, но чрезмерное применение этого совета может иметь непредвиденные последствия, если не будут предприняты надлежащие меры (более высокое октановое число топлива, более качественное движение смеси, сплав головки цилиндров и электроника, предназначенная для защиты двигателя). За исключением замены головок цилиндров и поршней, лучше всего начать с более толстых прокладок головки и работать с шлифовальной машиной на камерах сгорания с особым вниманием к острым краям.

    Посмотреть все 12 фотографий

    Увеличить перекрытие кулачков

    Эффективный распределительный вал с большей продолжительностью и подъемом — один из наших любимых способов повысить производительность двигателя, но иногда выбор кулачка может вызвать непредвиденные проблемы. Более распространенной проблемой является кулачок, который слишком велик для сжатия, из-за чего фазы газораспределения слишком агрессивны для статической степени сжатия, и происходит потеря нижнего предела. Однако иногда распределительный вал имеет широкий угол разделения лепестков, что может привести к задержке слишком большого количества заряда в цилиндре.Как узнать, слишком ли короткое перекрытие? Быстрый тест давления запуска скажет вам: все, что превышает 180 фунтов на квадратный дюйм, является признаком того, что вы находитесь в сфере гоночного двигателя с высокой степенью сжатия, которому может потребоваться гоночное топливо. Хорошее и безопасное значение для традиционного уличного бензинового двигателя составляет от 150 до 170 фунтов на квадратный дюйм. Если давление запуска слишком велико, вам понадобится кулачок, который задерживает меньше воздуха / топлива в камере сгорания.

    Посмотреть все 12 фотографий

    Улучшить движение смеси

    Все исправления, которые мы упомянули до сих пор, я называю «последующими» исправлениями, то есть они больше похожи на повязку, чем на настоящее лекарство.Я обнаружил, что большинство случаев детонации можно отнести к режиму горения, который искушает судьбу через поток через порт, вызывающий отделение топлива от воздуха. Движение воздуха и топлива через порт, клапанный карман и камеру сгорания является сложным, и если топливо не может равномерно смешиваться с воздухом в результате процесса, называемого завихрением (перекатывание в случае четырехклапанного двигателя), низкая производительность и детонация будет результатом. Гашение — еще одно связанное действие, которое происходит как раз в тот момент, когда поршень достигает ВМТ.Заряд, застрявший между поршнем и декой головки блока цилиндров, сдавливается в открытую часть камеры сгорания в последний момент перед воспламенением, давая заряду последний хороший шанс смешаться. Когда не происходит хорошей хореографии завихрения и гашения, возникает детонация. Единственное правильное лекарство — это набор головок цилиндров, включающий последние улучшения в движении смеси.

    Посмотреть все 12 фото

    Прямое впрыскивание: (не) будущее детонации

    Возможно, это скорее эпилог, чем лекарство от детонации, но оригинальные компоненты непрерывно работают над этой проблемой и добились невероятных успехов это доступно нам прямо сейчас.Новейшие двигатели, выходящие из Детройта (линейка силовых установок Ecoboost Ford с прямым впрыском топлива и Gen V LT1 от GM, чтобы назвать два), почти полностью исключили детонацию, поскольку топливо не добавлялось в уравнение до самого последнего момента. Это просто по концепции, если не по механической конструкции, но когда в цилиндре нет топлива, трудно иметь нежелательное преждевременное зажигание. В двигателях с прямым впрыском топливо впрыскивается непосредственно в камеру сгорания при давлении в тысячи фунтов на квадратный дюйм.В результате топливо может быть доставлено почти мгновенно и направлено в область поршня, которая не может обеспечить достаточно тепла для события предварительного воспламенения.

    Мы сильно упрощаем преимущества двигателя DI, которые выходят далеко за рамки сопротивления детонации, но легко понять, почему эти двигатели могут иметь повышенную степень сжатия, которая разрушила бы предыдущие, если бы они работали на обычном газовом насосе. Это факт, что мы быстро приближаемся к эре двигателей внутреннего сгорания, которые оставят детонацию в прошлом, но мы все еще должны понимать это для наших любимых старинных восьмицилиндровых двигателей!

    Просмотреть все 12 фото

    Причины и способы предотвращения

    Имеет несколько названий — стук, пинг, детонация и т. Д., И многие термины могут сделать мероприятие безобидным. По правде говоря, при умеренных и высоких нагрузках постоянное количество детонаций может вызвать катастрофический отказ двигателя, обычно в виде раздробленных подшипников штока, трещин в кольцевых зацепах или отверстия в поршне.

    Практически каждый двигатель с искровым зажиганием от средней до высокой мощности будет испытывать случайные детонации в течение всего срока службы. Это одна из тех вещей, которых нельзя полностью избежать, но их можно легко контролировать и удерживать в безопасных пределах с помощью датчиков и ответственной настройки.Для начала это помогает понять, что происходит внутри камеры сгорания, чтобы вызвать этот разрушительный свистящий звук.

    Что это такое
    В зависимости от нагрузки двигателя и настройки, свеча зажигания загорается в диапазоне от 45 градусов до 5 градусов перед верхней мертвой точкой (ВМТ) такта сжатия и воспламеняет топливно-воздушную смесь.

    Во время нормального цикла сгорания фронт пламени расширяется от точки воспламенения к стенкам цилиндра и головке поршня, этот процесс горения может занять до 90 градусов вращения коленчатого вала для полного сгорания.Детонация определяется как любое самовозгорание, возникающее после того, как процесс горения уже начался, и не зависит от начального фронта пламени. Это неконтролируемое событие может происходить из любого места в камере и обычно вызвано высокими температурами и / или давлением в цилиндре.

    Что он делает
    Теперь, когда у вас есть базовые представления о детонации и двух его основных причинах (тепло и давление), мы можем поговорить о связанных с ним разрушительных эффектах. Повреждение вызвано не энергией, высвобождаемой при детонации, а скорее скоростью высвобождения энергии (энергетический потенциал такой же, как и при нормальном цикле сгорания).Детонация часто считается эквивалентом удара по верхней части поршня взрывным молотком.

    Как это обнаружить

    Слева: Датчик детонации, обычно встречающийся на автомобилях EFI. Справа: электронные детекторы, обычно используемые тюнерами.


    При возникновении детонации можно услышать звуковой сигнал. В вашем среднем двигателе EFI обнаружение детонации зависит от использования одного или нескольких датчиков детонации, установленных в определенных местах на двигателе.Эти датчики представляют собой тип микрофона, который откалиброван для улавливания определенного диапазона частот, который, как известно, связан с детонацией. Когда датчик обнаруживает достаточно большое количество детонаций, ЭБУ начинает замедлять угол опережения зажигания или добавлять больше топлива, в зависимости от используемого ЭБУ.

    Эти частоты детонации будут различаться в зависимости от конструкции двигателя и также должны быть откалиброваны тюнером после модификации тяжелого двигателя.
    Австралийская компания по управлению двигателями Haltech создала отличное видео, объясняющее процесс калибровки:

    Профессиональные тюнеры часто полагаются на использование детонационных баллончиков (детонационных баллончиков) для обнаружения событий детонации в сильно модифицированных двигателях.Эти дет-банки могут быть как электронными, так и механическими, в первом из которых используется микрофон для передачи звука через пару наушников, а во втором — просто с помощью медного крепления и трубки для передачи звука, улавливаемого медью, на наушники. Бидоны Det также могут помочь ускорить процесс повторной калибровки датчика детонации.

    Как им управлять
    При настройке двигателя есть два основных источника тепла и давления: подача топлива и угол опережения зажигания.

    Момент зажигания для контроля давления
    При настройке угла опережения зажигания вы должны помнить о том, насколько вы опережаете время — большее время не всегда означает большую мощность.Идея состоит в том, чтобы рассчитать время искры в нужный момент до ВМТ, чтобы обеспечить достаточно времени горения для достижения пикового давления в цилиндре в оптимальной точке ВМТ.

    Заводская карта опережения зажигания от JDM Mitsubishi Evo X 2008 года выпуска (Degrees BTDC). Обратите внимание, что по мере увеличения нагрузки и числа оборотов опережение уменьшается. Приложения с наддувом имеют тенденцию работать с более низкой пиковой синхронизацией из-за повышенного давления в цилиндре, связанного с принудительной индукцией.

    Превышение опережения зажигания приведет к слишком раннему возникновению искры, в результате чего давление в цилиндре будет расти быстрее, чем может распространяться фронт пламени.Это создаст два источника давления в цилиндре, работающих одновременно (ход поршня и сгорание), в результате чего давление и температура в цилиндре превысят точку самовоспламенения несгоревшего топлива, все еще оставшегося в цилиндре, что мгновенно сгорит. Это самовозгорание является детонационным событием и является одной из наиболее частых причин выхода из строя поршня, штока и подшипника.

    Примеры отказов подшипников из-за детонации. Слева: усталость промежуточной футеровки на основе меди в трехметаллических подшипниках.Справа: Локальный чрезмерный износ из-за деформации шатуна от детонации.

    Заправка топливом для контроля температуры
    При настройке двигателя топливо используется как форма контроля температуры. Добавление большего количества топлива создает более богатую смесь и охлаждает камеру, а удаление топлива выжимает смесь и добавляет больше тепла.

    Haltech предлагает отличную аналогию, которая поможет вам понять этот процесс. «Думайте об этом как о выпечке торта. Когда вы закончите выпечку, откройте духовку и вытащите пирог, чтобы он остыл.Температура воздуха внутри духовки составляет 180 градусов по Цельсию, поэтому и торт, и стальная форма для выпечки имеют температуру 180 градусов, но при этом, когда вы кладете руки на 180 градусов воздуха, вы не обжигаетесь. Однако металлическая форма для торта наверняка обожжет руки, как и сам торт через пару секунд.

    Что вы хотите убрать из этого, так это то, что воздух — ужасный проводник тепла, потому что воздух на 180 градусов в духовке не обожжет вас, как форма для торта при той же температуре. Топливо намного лучше проводит тепло, поэтому, в общем, чем больше топлива вы добавляете, тем больше тепла будет отводиться от стенок цилиндров, поршней, клапанов и т. Д.

    Заводская топливная карта EDM Mitsubishi Evo X 2008 года (масштабируется для AFR). Обратите внимание, что по мере увеличения нагрузки и числа оборотов увеличивается заправка (богаче). Двигатели с наддувом обычно требуют как минимум на 10% более богатой топливной смеси для борьбы с детонацией, вызванной повышенными температурами цилиндров, создаваемыми принудительной индукцией.

    Однако можно добавить слишком много топлива, особенно в областях с низкой нагрузкой, таких как холостой ход или круиз, и вы можете вызвать стук или даже расплавить поршень, если вы слишком обеднены при более высоких нагрузках. Задача тюнеров — откалибровать несколько различных компонентов двигателя (MAF, VE, VVT, Boost, заправка, синхронизация зажигания и т. Д.) для достижения наиболее эффективных целей заправки и опережения зажигания для двигателя и его конкретных модификаций.

    Заключение
    Детонация может вызвать катастрофический отказ двигателя, если ее не остановить. Вот почему большинство современных двигателей включают в себя функцию отказоустойчивости в заводской настройке, чтобы замедлить опережение зажигания или добавить топлива, когда датчик детонации обнаруживает слишком большое количество детонаций. Чтобы предотвратить детонацию модифицированных двигателей, требуется настройка для корректировки заводских калибровок и приведения вашего двигателя в равновесие с вашими новыми модами.

    Детонация и предварительное зажигание — Savvy Aviation Resources

    Эти два аномальных явления горения, которые часто путают и неправильно понимают, столь же различны, как ночь и день.

    Майка Буша

    Хотя мы часто слышим, как люди описывают то, что происходит внутри цилиндров двигателя с циклом Отто, как взрыв, то есть насильственное, почти мгновенное событие, это не так. Воздушно-топливный заряд не взрывается при воспламенении от свечей зажигания, а скорее горит упорядоченным образом, начиная от свечей зажигания и продвигаясь по камере сгорания, пока он не гаснет при достижении стенок цилиндра и днища поршня в воздушно-топливном режиме. заряд полностью израсходован и гореть больше нечего.Событие возгорания занимает значительный период времени — примерно 6 миллисекунд или 90 ° вращения коленчатого вала, плюс-минус.

    Очень важно, чтобы пиковое давление происходило за пределами ВМТ, потому что геометрия коленчатого вала и шатуна около ВМТ не позволяет преобразовать давление сгорания в полезную работу (например, вращение коленчатого вала), а просто создает чрезмерное напряжение в цилиндре, поршне , шатун и коленчатый вал. На рисунке 2 делается попытка драматизировать этот момент.

    Детонация

    Но если процесс сгорания протекает слишком быстро и пик давления возникает слишком рано, результатом может быть избыточное давление, чрезмерные температуры и нестабильные импульсы давления, известные как «детонация».Это потому, что, когда поршень находится в непосредственной близости от ВМТ, он не может двигаться вниз в цилиндре, чтобы сбросить давление (и в процессе выполнить некоторую полезную работу). Неровный вид с зазубринами на верхнем следе на Рисунке 4 является характерным признаком давления детонации.

    В автомобиле мы обычно слышим детонацию в виде слышимого «стука». В самолете мы не можем — слишком много шума — но мы можем наблюдать его на мониторе двигателя в виде чрезмерного CHT и пониженного EGT.

    Детонация — это то, что происходит около точки пикового давления в событии сгорания, после того, как воздушно-топливный заряд нормально воспламеняется свечами зажигания. Он характеризуется аномальными скачками давления около точки пикового давления, вызванными самовозгоранием конечного газа из-за чрезмерной температуры и давления.

    Вопреки тому, что вам сказали CFI или A&P, детонация не обязательно опасна. Многие двигатели довольно регулярно работают в режиме легкой детонации, а некоторые могут выдерживать умеренную детонацию в течение продолжительных периодов времени без повреждений.Детонация — не оптимальная ситуация, но она не обязательно разрушительна. Чем выше удельная мощность двигателя, тем больше вероятность, что он получит повреждения от детонации. Двигатель, который производит 0,5 л.с. / дюйм3 (мощность на кубический дюйм рабочего объема) — как это типично для большинства авиационных двигателей с карбюратором — обычно может выдерживать умеренные уровни детонации без повреждений, но двигатели с турбонаддувом с большим наддувом и мощностью 0,625 л.с. / дюйм3 или более могут быть довольно быстро повреждены детонацией.

    Когда происходит детонационное повреждение, оно обычно проявляется в виде трещин (электродов и изоляторов свечей зажигания, а иногда и поршневых колец и площадок), точечной коррозии (обычно головки поршня) и / или теплового повреждения (часто юбки поршня). задиров и оплавление углов поршня).

    Как пилоты, мы обычно можем избежать такого повреждения, если будем предупреждать о чрезмерном CHT и пониженном EGT, которые характерны для детонации, и быстро реагируем снижением мощности и переходом на полностью обогащенную смесь. Здесь очень важен контроль двигателя — иначе вы не сможете увидеть CHT пяти из шести цилиндров, а программирование сигнала CHT на срабатывание при 400 ° поможет привлечь ваше внимание и предпринять соответствующие действия.

    Предварительное зажигание

    «Предварительное зажигание» — это еще одно событие ненормального горения, которое часто путают с детонацией, но на самом деле это совершенно другое.Предварительное зажигание — это зажигание топливовоздушной смеси перед зажиганием свечи зажигания. Каждый раз, когда что-то вызывает воспламенение смеси в камере до возгорания свечей зажигания, это классифицируется как преждевременное зажигание. Источником воспламенения может быть перегретый наконечник свечи зажигания, нагар или свинец в камере сгорания или (редко) сгоревший выпускной клапан — любая из этих вещей может действовать как свеча накаливания, преждевременно воспламеняя заряд.

    Такое горячее пятно в камере может воспламенить заряд, в то время как поршень находится на очень ранней стадии сжатия.Результат: значительную часть всего такта сжатия двигатель пытается сжать горячую массу расширяющегося газа. Это, очевидно, создает огромную механическую нагрузку на двигатель и передает большое количество тепла алюминиевой головке поршня и головке блока цилиндров. Существенный ущерб почти неизбежен.

    Детонация вызывает очень быстрый скачок давления около точки пикового давления на очень короткий период времени. Предварительное зажигание вызывает огромное давление, которое присутствует в течение очень долгого времени — возможно, на всем такте сжатия.Мало того, что преждевременное возгорание гораздо опаснее, его гораздо труднее обнаружить. Фактически, обычно вы узнаете об этом только после того, как двигатель будет катастрофически поврежден.

    Двигатели могут выдерживать детонацию в течение значительных периодов времени, но не существует двигателя, который мог бы выжить очень долго при преждевременном воспламенении. Двигатель не будет работать более нескольких секунд с предварительным зажиганием. Если вы видите коронку поршня, которая выглядит обработанной пескоструйным аппаратом, или трещину на кольце, вероятно, это было вызвано сильным детоатом.Если вы видите расплавленное отверстие в середине днища поршня, это, вероятно, было вызвано преждевременным зажиганием. Другими признаками преждевременного воспламенения являются свечи зажигания с расплавленными электродами или изоляторы, забрызганные расплавленным металлом. На рис. 5 показан пример серьезных повреждений, вызванных преждевременным зажиганием.

    Предварительное зажигание, вызванное детонацией

    Хотя детонация и преждевременное зажигание — два совершенно разных явления, сильная детонация может вызвать преждевременное зажигание. Если двигатель работает в режиме сильной детонации в течение значительного периода времени, чрезмерные температуры и скачки давления (которые нарушают обычный защитный пограничный слой) могут вызвать перегрев электродов свечей зажигания и других предметов в камере сгорания до точки, при которой они запустятся. раскалиться докрасна.В этот момент светящийся предмет может вызвать преждевременное зажигание и быстрое разрушение цилиндра. После разборки судебно-медицинский анализ выявит явные признаки как детонационного, так и предварительного воспламенения повреждений, хотя в конечном итоге двигатель сработал именно из-за предварительного воспламенения.

    В другой статье мы более подробно рассмотрим процесс нормального сгорания и исследуем, как использование нами элементов управления двигателем — дроссельной заслонки, смеси и пропуска — влияет на то, что происходит внутри цилиндра.

    © 2007-2013 — Майкл Д.Busch — Все права защищены.

    Предварительное зажигание, детонация и детонация

    Детонация
    Детонация — это неконтролируемое сгорание конечных газов в цилиндре и, по определению, всегда происходит после искрового зажигания (в отличие от искрового зажигания, как в случае с предварительным зажиганием). Это происходит, когда в топливе не хватает октанового числа, чтобы противостоять неконтролируемому сгоранию для свойств двигателя, в котором оно используется, но это также может быть вызвано чрезмерно бедной топливной смесью.

    Если форма камеры сгорания двигателя не подходит для чрезвычайно быстрого распространения пламени и используется топливо с более низким, чем необходимо, октановым числом, накопление тепловых волн и волн давления в цилиндре при сгорании топливной смеси могут воспламениться концевые газы в цилиндре (детонация) и, таким образом, вызвать дополнительные волны тепла и давления, которые могут потенциально разрушить двигатель. Двигатель может допускать детонацию, если она не является серьезной по сравнению с конструкцией двигателя, но она может быть чрезвычайно разрушительной, если она достаточно серьезна.Что касается самого двигателя, одно из ключевых различий между детонацией и предварительным зажиганием заключается в том, что двигатель может быть спроектирован и построен таким образом, чтобы выдерживать детонацию от легкой до умеренной, но не обязательно с предварительным зажиганием. Однако ни то, ни другое не желательно, поскольку не способствует стабильности и контролю горения. Чтобы упростить, если вы планируете интенсивно эксплуатировать свой двигатель, используйте свечу зажигания с подходящим тепловым диапазоном для вашего двигателя, не запускайте чрезмерно богатую или бедную топливную смесь, проверяйте и часто меняйте свечи и используйте топливо самого высокого качества. может позволить себе соответствующее октановое число для вашего двигателя (подробнее о характеристиках топлива и октановом числе мы расскажем в следующей статье).

    Если вы настроили свой двигатель и скорректировали характеристики моделирования воздушного потока, правильно настроив кривую MAF и / или сопоставив таблицу VE для достижения желаемой топливной смеси, установите значения обогащения мощности в WOT на значение, которое приведет к с максимальной выходной мощностью (MBT) без ущерба для срока службы двигателя, которого вы хотите достичь. Например, большинство серийных двигателей (при условии использования неэтилированного бензина премиум-класса), таких как двигатели GM LS, модульные двигатели Ford и двигатели Coyote и двигатели Chrysler / Dodge Hemi, имеют соотношение воздух / топливо 12.8: 1 действительно хорошо работает как компромисс между большой выходной мощностью и долгим сроком службы. Если в качестве приоритета выбрана выходная мощность, вы можете работать на обедненной смеси 13,2: 1 (если динамометрические тесты показывают прирост мощности) без остановки двигателя, однако вы будете нагружать двигатель значительно сильнее, чем при AFR 12,8: 1. Для более тяжелых транспортных средств (более 3800 фунтов) может потребоваться более богатая смесь, такая как AFR 12,5: 1, чтобы свести детонацию к минимуму при WOT. При более низком передаточном числе задней оси (3,55: 1 и численно выше) вы можете снова попробовать наклониться и можете обнаружить, что детонация не происходит.Работа с обедненной смесью, превышающей 13,2: 1, может привести к увеличению мощности двигателя (и на самом деле некоторые двигатели могут лучше всего работать на этих более бедных топливных смесях), но помните, что работа со слишком бедной смесью может вызвать чрезмерный нагрев, износ и увеличить вероятность преждевременного воспламенения. Всякий раз, когда происходит детонация, у вас есть несколько вариантов (при условии, что вы придерживаетесь рассматриваемого топлива): Вариант 1 — обогатить топливную смесь и провести испытания, чтобы убедиться, что детонация предотвращена без потери мощности. Вариант 2 — установить меньшую синхронизацию опережения зажигания в основной карте зажигания.Вариант 3 — терпеть детонацию, если она небольшая. Вариант 3 может работать, если датчик детонации улавливает только небольшую детонацию и заставляет ЭБУ замедлить синхронизацию не более чем на 2-4 градуса. Если вы занимаетесь драг-рейсингом, это может сработать для вас. Если вы занимаетесь шоссейными гонками, избегайте Варианта 3 и придерживайтесь Варианта 1, а в случае неудачи переходите к Варианту 2. Если двигатель работает в режиме WOT в течение длительного времени (подумайте о длинных прямых для гонок на выносливость, таких как Daytona, Sebring или Road America), вы можете подумать о создании отдельной калибровки двигателя специально для этих гусениц, чтобы работать с немного более богатой топливной смесью на WOT и, при необходимости, на пару градусов меньше опережения зажигания, чтобы быть уверенным, что двигатель благополучно выживает. на WOT во время этих гонок на выносливость с тяжелым газом.Внесение этих изменений приводит только к потере нескольких лошадиных сил по сравнению с более радикальным маршрутом настройки, поэтому, если вы не гонитесь за серьезной суммой денег, это может быть разумным курсом действий. После внесения этих изменений в калибровку ЭБУ не забудьте еще раз проверить свечи зажигания, чтобы убедиться, что вы используете соответствующий диапазон нагрева для новой настройки.

    Детонация | lycoming.com

    Что такое детонация?

    Детонация — это резкое сгорание или взрыв топливного заряда внутри цилиндра.Во время нормального сгорания свечи зажигания воспламеняют топливный заряд, и топливо имеет постоянное и равномерное горение, поскольку поршень проходит рабочий такт, и химическая энергия эффективно преобразуется в механическую. Проще говоря, когда происходит детонация, топливный заряд быстро воспламеняется в результате неконтролируемого взрыва, вызывая ударную силу на поршень, а не постоянный толчок. Легкая детонация может не показывать никаких признаков в салоне самолета. Детонация от умеренной до сильной может быть замечена по неровности двигателя, вибрации или потере мощности и, в конечном итоге, к повреждению двигателя.Пилоту всегда следует следить за неожиданно высокими температурами головки цилиндров (CHT) или выхлопными газами (EGT), которые могут быть признаком того, что происходит детонация.

    Что вызывает детонацию и как ее предотвратить?

    Процесс сгорания внутри поршневого двигателя довольно динамичный, и есть много вещей, которые могут способствовать детонации. В этой статье мы коснемся нескольких наиболее распространенных причин, а не краткого списка.

    Во-первых, давайте предположим, что самолет и двигатель заправлены топливом правильно и что октановое число топлива соответствует или превышает октановую потребность двигателя.Инструкция по обслуживанию Lycoming 1070 содержит исчерпывающий перечень одобренных видов топлива для наших двигателей, а также другую важную информацию.

    Учитывая, что топливо является правильным выбором для двигателя, для пилота причиной номер один детонации является чрезмерная обедненная смесь при высоких настройках мощности. Пилот всегда должен придерживаться указаний в утвержденном руководстве по эксплуатации для правильных настроек наклона и мощности. Чтобы ознакомиться с рекомендациями Lycoming, обратитесь к текущим редакциям соответствующего руководства оператора Lycoming и инструкции по обслуживанию 1094.Если пилот считает, что двигатель может взорваться, он или она может предпринять следующие действия.

    • Увеличить моторную смесь.
    • Уменьшите мощность до более низкого значения.
    • Уменьшите или прекратите набор высоты и увеличьте скорость движения для лучшего охлаждения.

    Для механика причиной номер один детонации будет любая проблема, которая может привести к неожиданной обедненной работе цилиндра. Чаще всего это вызвано частичным засорением форсунки для впрыска топлива или утечкой всасываемого воздуха.Каждый раз, когда топливные форсунки снимаются, их следует прочистить и проверить поток. Во время осмотра механик должен искать признаки утечки на впуске; обычно отмечается синим пятном топлива на впускных трубах. Перед дальнейшим полетом следует исправить любые аномалии.

    Мы также видели случаи, когда треснувшие или иным образом поврежденные свечи зажигания создавали «горячую точку» в двигателе, и происходила детонация. Вот почему никогда не стоит использовать вилку, которая упала на твердый пол или была повреждена иным образом.

    Двигатели

    Lycoming соответствуют требованиям FAA по запасу детонации или превосходят их. Следовательно, если двигатель обслуживается и эксплуатируется в соответствии с нашими опубликованными инструкциями, двигатель никогда не должен взрываться.

    Как мой механик или мастерская по ремонту двигателей узнает о взрыве?

    Детонация оказывает негативное воздействие на весь двигатель. Легкая детонация может вызвать преждевременный износ подшипников и втулок. Сильная или продолжительная детонация может вызвать повреждение головки цилиндров и поршней.В некоторых крайних случаях шатун может быть погнут или сломан, головка цилиндра может треснуть или выйти из строя, или могут сломаться площадки поршневого кольца.

    Каждый раз, когда цилиндр снимается, ваш механик должен воспользоваться возможностью, чтобы проверить цилиндр и поршни на наличие признаков неисправности. Вот некоторые вещи, которые можно проверить.

    • Хотя это может выглядеть не лучшим образом, накопление свинца или отложения сгорания являются нормальным явлением для двигателей Lycoming. Отсутствие этих депозитов — тоже не обязательно хорошо.Необходимо проверить головку блока цилиндров и поршень на предмет «пескоструйной обработки». Отсутствие отложений или чистая головка и поверхность поршня могут указывать на детонацию. При использовании неэтилированного топлива отложения должны быть…
    • Повреждение от детонации обычно проявляется на краях поршней и на головке цилиндров между портами свечей зажигания и клапанами.

    По вопросам ухода за двигателем Lycoming и его обслуживания обращайтесь в нашу службу технической поддержки по адресу: Technicalsupport @ lycoming.com или по телефону + 1-800-258-3279.

    Детонация против предварительного воспламенения

    Есть много способов испортить отличный двигатель, но сегодня я просто хочу поговорить о двух из самых жестоких. Детонация и предварительное зажигание, которые часто меняются местами и / или используются для описания одного и того же, на самом деле являются совершенно разными вещами, которые приводят к одинаковым результатам. Оба они называются ненормальным сгоранием, и они очень вредны для вашего двигателя. Чтобы лучше объяснить как детонацию, так и предварительное зажигание, мне нужно также объяснить нормальное горение.

    Нормальное сгорание:

    Нормальное горение — это горение топливовоздушной смеси в камере сгорания. Нормальное сгорание начинается с того, что фронт пламени возникает у свечи зажигания и равномерно и равномерно распространяется наружу по камере сгорания. Это очень похоже на надувание воздушного шара. Когда вы дуетесь, воздушный шар очень контролируемо и равномерно расширяется от источника воздуха. В идеальном мире сгорание сжигает весь воздух и топливо в цилиндре, не оставляя никого позади (это происходит со стехиометрической смесью лямбда-1).Тепло от сгорания передается от фронта пламени к поршню, от поршня к стенке цилиндра и оттуда в систему охлаждающей жидкости. Распространенное заблуждение о горении — это взрыв. Это просто неправда … В идеале, когда свеча зажигания зажигает смесь, пламя заполняет цилиндр очень быстро, но очень контролируемым образом.

    Детонация:

    Определение: Самовозгорание отходящего газа или оставшейся топливно-воздушной смеси в камере.

    Детонация всегда возникает после того, как свеча зажигания начала нормальное сгорание. Обычное горение расширяется, но газы на краю фронта пламени сжимаются и начинают спонтанно воспламеняться. Вероятно, это вызвано чрезмерным нагревом и давлением. Однако самое важное, что нужно помнить о детонации, — это то, что она возникает после того, как фронт пламени был инициирован свечой зажигания.

    Есть много факторов, которые вместе создают идеальный сценарий взрыва.Хотя конструкция двигателя и октановое число топлива играют важную роль, наиболее распространенным причинным фактором является слишком большой опережение зажигания. Чрезмерно опережающий угол опережения зажигания приводит к слишком быстрому зажиганию, что приводит к слишком быстрому увеличению давления. Это очень высокий / очень резкий скачок давления, который часто приводит к повреждению двигателя.

    Как вы можете видеть на изображении, график вверху имеет плавный профиль давления и может считаться нормальным сгоранием. Однако на нижнем графике показано нормальное повышение давления до тех пор, пока детонация не произойдет даже после искры.Затем вы видите большой скачок давления из-за ненормального сгорания. Этот скачок давления заставляет структуру двигателя резонировать, как будто это камертон. Этот резонанс улавливается датчиком детонации и передается в ЭБУ.

    Датчики детонации

    — большой повод для беспокойства у многих энтузиастов. Возможность видеть то, что они видят с помощью устройства мониторинга, такого как Cobb Tuning AccessPort, дает людям возможность всегда видеть, что происходит с их движком. Это окно позволяет вам увидеть то, что вы обычно никогда не замечаете или о чем не заботитесь.Меня очень часто спрашивают о продолжающейся детонации, которая происходит при частичном открытии дроссельной заслонки. К счастью, детонация не всегда разрушительна. Низкие уровни детонации возникают постоянно и даже могут поддерживаться в течение длительных периодов времени без нанесения ущерба. Частичный стук дроссельной заслонки является нормальным явлением для различных автомобилей и, хотя иногда он может быть вызван фактической детонацией, в большинстве случаев это просто шум, поскольку двигатель резонирует при определенных оборотах в минуту. Это также будет проявляться в точках переключения передач, когда двигатель значительно перемещается между переключениями передач на WOT, и это не должно вызывать беспокойства.Однако детонация становится серьезным поводом для беспокойства, когда вы начинаете работать с более высокими нагрузками. Если вы видите значительный стук в широко открытой дроссельной заслонке (WOT), вам следует обратиться к своему тюнеру.

    Повреждение: Есть несколько основных неисправностей, вызванных детонацией. Меньшей из точек неисправности является точечная коррозия или истирание днища поршня. Вы также увидите эту точечную коррозию на выпускных клапанах, так как это более горячая сторона цилиндра, при этом воздушно-топливная смесь охлаждает сторону впуска.Ямка выглядит так, как будто в поршень попала птица, выпущенная из дробовика.

    Еще одна точка повреждения от детонации — это приземление кольца. Часто под резкими скачками давления вы получаете раздавленные или сломанные кольцевые площадки. В менее тяжелых случаях вы все равно увидите разорванные кольца. Это часто случается с литыми поршнями, поскольку они никогда не были предназначены для того, чтобы выдерживать такое давление в цилиндре, особенно такие внезапные и резкие изменения давления.


    Вместе с детонацией приходит тепло.Скачки давления разрушают пограничный слой газа, который гасит фронт пламени и защищает относительно холодный поршень от относительно горячего сгорания. По мере того как этот пограничный слой разрушается и все больше и больше тепла поглощается поршнем, вы увидите деформацию поршня и задиры на стенках цилиндра, что неизбежно приведет к необходимости ремонта двигателя. Из-за этого вы также увидите более высокие температуры охлаждающей жидкости, поскольку системе охлаждающей жидкости приходится выполнять больше работы с избыточным теплом.

    Индикаторы: Слышны более высокие уровни детонации, они будут звучать так, как будто четвертаки стучат по стеклу. Вы не можете услышать его в большинстве новых автомобилей из-за изолированной кабины, поэтому, когда вы его слышите, это, скорее всего, более высокий уровень стука. Если у вас есть устройство настройки, которое контролирует замедление детонации, такое как Cobb Tuning Accesport, вы видите только реакцию двигателя на воспринимаемый шум. Однако стоит обратить внимание на то, что двигатель по какой-то причине пытается спастись за счет уменьшения опережения зажигания.

    Предварительное зажигание:

    Определение: Предварительное зажигание — это воспламенение топливно-воздушной смеси перед воспламенением свечи зажигания.

    Событие преждевременного воспламенения выглядит примерно так…

    Топливо-воздушная смесь поступает в камеру сгорания, когда поршень находится на такте впуска вниз. Затем поршень возвращается вверх для такта сжатия. Чем более сжатая смесь, тем труднее ее зажечь, поэтому, когда поршень находится на нижней стороне такта сжатия, смесь фактически легче воспламеняется, чем когда она приближается к верхней мертвой точке (ВМТ).Горячее пятно в цилиндре, такое как раскаленный наконечник свечи зажигания, может воспламенить эту смесь с низким уровнем сжатия очень рано, до того, как загорится свеча зажигания. Теперь движение поршня вверх борется с расширяющейся силой сгорания. Это добавляет огромное количество тепла и нагрузки к двигателю, и по этой причине предварительное зажигание отвечает за гораздо более высокое давление в цилиндре, чем детонация. Давление от предварительного зажигания не такое быстрое, как при детонации. Напротив, давление очень высокое и длится гораздо дольше.

    Повреждение: Повреждение от предварительного воспламенения намного более серьезное и мгновенное, чем от детонации. Как правило, при предварительном зажигании вы увидите расплавленные отверстия в поршнях, расплавленные свечи зажигания, а отказ двигателя происходит практически сразу.

    Из-за большей продолжительности нагрева и давления, вызванных предварительным зажиганием, вы заметите намного больше расплавленных деталей, тогда как при детонации вы получите больше деталей, которые просто разнесутся на части.

    Индикаторы: На самом деле нет никаких предупреждающих знаков с предварительным зажиганием.Лучшее, что вы можете сделать, чтобы предотвратить это, — это убедиться, что двигатель настроен, а также минимизировать возможные горячие точки.